SEARCH (Type the topic here)

Stomach and Esophagus MCQ

1. Which of the following statements about the anatomic course of the esophagus is correct?
A. The cervical esophagus passes behind and to the right of the trachea.
B. The thoracic esophagus enters the posterior mediastinum anterior to the aortic arch.
C. The thoracic esophagus passes behind the right mainstem bronchus and the pericardium.
D. The esophagus enters the diaphragmatic hiatus at the level of T8.
E. The esophagus deviates anteriorly and to the left as it enters the abdomen.
Answer: E

DISCUSSION: Knowledge of the normal course of the esophagus is important in operative and endoscopic procedures. The cervical esophagus lies just anterior to the prevertebral fascia and courses through the neck
posterior to the trachea and to the left of the midline. The cervical esophagus is therefore more readily approached surgically through a left neck incision. The thoracic esophagus enters the posterior mediastinum posterior to the aortic arch. Therefore, when operating on the upper esophagus, a right thoracotomy is preferred, since the aortic arch overlies the esophagus in the left chest. The esophagus descends in the posterior mediastinum behind the left mainstem bronchus and pericardium. It is the proximity of the esophagus and left mainstem bronchus that is responsible for the development of malignant tracheoesophageal fistulas between these two structures. The esophagus enters the diaphragmatic hiatus, which is located at the level of T11. As it enters the abdomen, the esophagus deviates anteriorly and to the left. Therefore, when performing rigid esophagoscopy as the distal esophagus is entered, the patient's head must be turned to the right and the esophagoscope elevated anteriorly to avoid perforation.


2. Which of the following statements about esophageal anatomy is correct?
A. The esophagus has a poor blood supply, which is segmental in distribution and accounts for the high incidence of anastomotic leakage.
B. The esophageal serosa consists of a thin layer of fibroareolar tissue.
C. The esophagus has two distinct muscle layers, an outer, longitudinal one and an inner, circular one, which are striated in the upper third and smooth in the distal two thirds.
D. Injury to the recurrent laryngeal nerve results in vocal cord dysfunction but does not affect swallowing.
E. The lymphatic drainage of the esophagus is relatively sparse, localized primarily to adjacent paraesophageal lymph nodes.
Answer: C

DISCUSSION: Poor technique, not poor blood supply, explains most esophageal anastomotic leaks. While the major blood supply of the esophagus is from four to six segmental aortic esophageal arteries, there are extensive submucosal collaterals from the inferior thyroid, intercostal, bronchial, inferior phrenic, and left gastric arteries. The esophagus lacks serosa and instead is surrounded by mediastinal connective tissue (adventitia). There are two muscle layers in the esophagus, an outer longitudinal and an inner circular one. Both layers of the upper third of the esophagus consist of striated muscle, while in the lower two thirds they are (nonstriated) smooth muscle. The recurrent laryngeal branches of the vagus nerves provide both parasympathetic innervation to the cervical esophagus and innervation to the upper esophageal sphincter (UES). Injury to the recurrent laryngeal nerve therefore results in improved UES function with secondary aspiration on swallowing as well as vocal cord dysfunction and hoarseness. The esophagus has extensive lymphatic drainage, with lymphatic capillaries coursing longitudinally in the esophageal wall and communicating with paraesophageal, paratracheal and subcarinal, other mediastinal, perigastric, and internal jugular lymph nodes. This accounts for the biologically aggressive nature of esophageal carcinoma, which tends to metastasize early in its course.


3. Which of the following statements about the lower esophageal sphincter (LES) mechanism, or high-pressure zone (HPZ), is true?
A. The LES is a circular smooth muscle ring that is 3 to 5 cm. long.
B. In assessing esophageal manometric data, mean HPZ pressure less than 6 mm. Hg or overall length less than 2 cm. is more likely to be associated with incompetence of the LES and gastroesophageal reflux.
C. Esophageal manometry and the acid perfusion (Bernstein) test reliably identify the patient with an incompetent LES mechanism.
D. Distal HPZ relaxation occurs within 5 to 8 seconds of initiating a swallow.
E. Twenty-four–hour distal esophageal pH monitoring is achieved with an intraesophageal pH electrode positioned at the esophagogastric junction.
Answer: B

DISCUSSION: The LES is not an anatomic muscular sphincter like the anus or pylorus, but rather, is a functional sphincter that serves as a barrier against abnormal regurgitation of gastric contents into the esophagus and is more appropriately termed the distal esophageal HPZ. While, in general, no HPZ value absolutely indicates either competence or incompetence of the LES mechanism, patients with a mean HPZ pressure less than 6 mm. Hg or a sphincter length less than 2 cm. are likely to have an incompetent LES and gastroesophageal reflux. Esophageal manometry defines the amplitude and length of the distal HPZ and the character of esophageal peristalsis. It does not determine whether or not the LES is competent. The Bernstein test identifies the patient with an acid-sensitive esophagus but does not indicate whether or not the patient has gastroesophageal reflux or esophagitis. Reflex distal HPZ relaxation occurs within 1.5 to 2.5 seconds after a swallow is initiated and lasts 4 to 6 seconds. Twenty-four–hour distal esophageal pH monitoring is achieved by means of an intraesophageal pH electrode positioned 5 cm. proximal to the HPZ, as determined by prior manometric evaluation.


4. Which of the following statements about esophageal motility is/are true?
A. The act of swallowing initiates UES relaxation, which persists until the bolus of food passes the LES.
B. The primary peristaltic wave normally propels the swallowed bolus through the esophagus in 4 to 8 seconds.
C. Normally, a progressive peristaltic contraction (primary wave) follows 50% of all swallows, the remainder being secondary or tertiary contractions.
D. Secondary peristalsis is initiated when the entire swallowed bolus of food fails to empty from the esophagus into the stomach.
E. Tertiary esophageal contractions are high-amplitude progressive peristaltic contractions that produce the “corkscrew” appearance of esophageal spasm on barium esophagography.
Answer: BD

DISCUSSION: Swallowing initiates UES relaxation, which lasts only 0.5 to 1.0 second, considerably less than the 4 to 8 seconds required for a swallowed bolus to pass through the esophagus and into the stomach. A progressive peristaltic contraction normally follows 97% of all swallows. If the swallowed bolus does not empty completely into the stomach, distention of the esophagus initiates secondary peristalsis, which is progressive and sequential and persists until the retained intraesophageal contents have passed into the stomach. Tertiary contractions are simultaneous, nonprogressive, incoordinated contractions of the esophageal smooth muscle that produce the contraction rings responsible for the corkscrew esophagus of diffuse esophageal spasm on barium esophagography.


5. Which of the following statements about UES dysfunction are correct?
A. This condition is diagnosed by the characteristic manometric findings of UES spasm.
B. Typical symptoms include cervical dysphagia, expectoration of saliva, and hoarseness.
C. The classic finding on barium esophagogram is a posterior cricopharyngeal bar.
D. Medical or surgical therapy of gastroesophageal reflux may be curative.
E. A cervical esophagomyotomy for UES dysfunction should be limited to 2 to 3 cm. in length so that normal muscle is not damaged.
Answer: BCD

DISCUSSION: The unique anatomic characteristics of the UES and the limitations of existing equipment in recording the rapid sequence of events associated with swallowing make standard manometric definition of UES motor abnormalities extremely difficult. Characteristic consistent abnormalities of UES function in patients with cricopharyngeal dysfunction have not been well documented. UES dysfunction results in cervical dysphagia, expectoration of saliva that is no longer swallowed freely, and, often, intermittent hoarseness due to alteration of the larynx and vocal cords by the pull of the abnormal cricopharyngeal sphincter. Barium esophagography in the patient with UES dysfunction frequently shows a posterior cricopharyngeal “bar,” representing the prominent impression of the sphincter on the esophageal lumen. Patients with gastroesophageal reflux may present with cervical dysphagia due to secondary UES dysfunction. Successful medical or surgical treatment of the reflux may eliminate the cervical complaints. Since the UES is normally 3 to 5 cm. long, when a cervical esophagomyotomy is required for treatment of cricopharyngeal dysfunction, a generous myotomy, 7 to 10 cm. long, is carried out to ensure complete division of all incoordinated UES muscle fibers.


6. Which of the following statements about achalasia is/are correct?
A. In most cases in North America the cause is a parasitic infestation by Trypanosoma cruzi.
B. Chest pain and regurgitation are the usual symptoms.
C. Distal-third esophageal adenocarcinomas may occur in as many as 20% of patients within 10 years of diagnosis.
D. Manometry demonstrates failure of LES relaxation on swallowing and absent or weak simultaneous contractions in the esophageal body after swallowing.
E. Endoscopic botulinum toxin injection of the LES, pneumatic dilatation, and esophagomyotomy provide highly effective curative therapy for achalasia.
Answer: D

DISCUSSION: While in South America achalasia is the result of Chagas' disease caused by parasitic infestation by the leishmanial forms of T. cruzi, in Europe and North America the cause of achalasia is unknown. The common presenting symptoms of achalasia are dysphagia, regurgitation, and weight loss. Chest pain is an infrequent symptom in achalasia and is more characteristic of esophageal spasm. Achalasia is a premalignant esophageal lesion: the retention esophagitis leads to metaplasia and squamous cell carcinoma, which occurs after 15 to 25 years in the middle third of the thoracic esophagus in 10% of patients. The classic manometric findings of achalasia are failure of relaxation of the LES on swallowing and absent or weak simultaneous contractions in the esophageal body after swallowing. Achalasia is currently incurable, and, though the recently described endoscopic botulinum toxin injection of the lower esophageal sphincter, pneumatic dilatation, and esophagomyotomy effectively relieve dysphagia in the majority of patients, all of these treatments are strictly palliative. The motility disturbance persists throughout life.


7. Which of the following statements about diffuse esophageal spasm is/are true?
A. Chest pain due to esophageal spasm is readily differentiated from angina pectoris of cardiac origin.
B. Bouts of esophageal obstruction and regurgitation of food are characteristic.
C. Associated psychiatric disorders are common.
D. During manometric assessment, unless the patient is having pain there may be no detectable multiphasic, high-amplitude, simultaneous esophageal contractions.
E. The treatment of choice is a long esophagomyotomy from the aortic arch to the esophagogastric junction.
Answer: CD

DISCUSSION: The chest pain of diffuse esophageal spasm is often indistinguishable from that of angina pectoris of cardiac origin. Many patients undergo cardiac catheterization to rule out coronary artery disease. Patients may experience slow emptying of the esophagus, but obstructive symptoms and regurgitation of food are unusual. Psychiatric disorders (depression, psychosomatic complaints, anxiety) have been documented in more than 80% of patients with esophageal contraction abnormalities. The classic manometric criteria of diffuse esophageal spasm are simultaneous, multiphasic, repetitive, high-amplitude contractions occurring after a swallow and spontaneously. These changes may not be detected if manometry is performed when the patient is having no pain. As the cause of esophageal spasm is unknown, treatment is far from ideal. Conservative management—avoidance of “trigger” foods and drinks, psychiatric counseling, treatment of reflux, esophageal dilatations, use of histamine H 2 blockers, anti-spasmodics, and smooth muscle relaxants—should always be attempted first. Esophagomyotomy provides no lasting relief of esophageal spasm for as many as 50% of patients and should be used only in a minority of these patients.


8. Which of the following statements about epiphrenic diverticula of the esophagus is/are correct?
A. They are traction diverticula that arise close to the tracheobronchial tree.
B. They characteristically arise proximal to an esophageal reflux stricture.
C. The degree of dysphagia correlates with the size of the pouch.
D. They are best approached surgically through a right thoracotomy.
E. The operation of choice is a stapled diverticulectomy, long esophagomyotomy, and partial fundoplication.
Answer: E

DISCUSSION: Epiphrenic diverticula are pulsion diverticula that arise in the distal 10 cm. of the esophagus. The cause is elevated intraesophageal pressure, which forces mucosa and submucosa to herniate through the muscle layers. Though this may occur with an esophageal reflux stricture or tumor, primary disordered esophageal motility is the most common cause. Many patients are asymptomatic when their epiphrenic diverticula are diagnosed by barium esophagography, symptoms being related more to the degree of disordered motility than to the size of the pouch. Distal esophageal diverticulum is best approached through a left thoracotomy. When surgery is indicated, the preferred approach is transthoracic diverticulectomy using a surgical stapler, a long esophagomyotomy to relieve the elevated intraesophageal pressure, and a nonobstructing partial (e.g., Belsey's) fundoplication.


9. Which of the following statements about Schatzki's ring is correct?
A. The ring represents a panmural fibrotic stricture resulting from gastroesophageal reflux.
B. Dysphagia occurs when the ring diameter is 13 mm. or less.
C. The ring occurs within 1 to 2 cm. of the squamocolumnar epithelial junction.
D. Schatzki's ring indicates reflux esophagitis.
E. Schatzki's ring signifies the need for an antireflux operation.
Answer: B

DISCUSSION: Schatzki's ring is seen radiographically as an annular weblike constriction at the esophagogastric junction in a patient with a sliding hiatalhernia. It represents prominence of the esophagogastric junction with slight submucosal fibrosis but not true panmural fibrotic reflux stricture. Intermittent dysphagia may occur when the ring size is 20 mm. or less, but dysphagia is almost invariable when the ring measures 13 mm. or less. Schatzki's ring occurs precisely at the squamocolumnar epithelial junction. It is indicative of the presence of a hiatal hernia but not of gastroesophageal reflux or esophagitis. An asymptomatic Schatzki's ring requires no therapy per se. Patients with refractory severe reflux symptoms after dilation therapy are candidates for antireflux surgery.


10. Choose the distance in centimeters from the upper incisor teeth at which the following radiographically identified esophageal lesions would be encountered endoscopically:
A. 10 cm. 1. Zenker's diverticulum
B. 15 cm. 2. Traction diverticulum
C. 25 cm. 3. Tumor 10 cm. proximal to the esophagogastric junction
D. 30 cm.
E. 40 cm.
Answer: 1-B, 2-C, 3-D

DISCUSSION: A barium esophagogram should be obtained routinely before performing elective esophagoscopy. The location of an esophageal lesion seen on the barium swallow study can be related to adjacent anatomic landmarks. This allows the endoscopist to anticipate the level (as measured from the upper incisor teeth) where he should expect to see the abnormality at esophagoscopy. For example, the upper esophageal (cricopharyngeal) sphincter is generally seen at the level of the C7–T1 vertebrae radiographically and at 15 cm. endoscopically. This is the level at which the mouth of a Zenker's diverticulum is seen. The tracheal bifurcation occurs at the level of the T4 or 25 cm. from the upper incisors endoscopically; a traction (parabronchial) diverticulum at the level of the carina on a barium esophagogram will be seen at approximately 25 cm. The esophagogastric junction occurs at approximately the level of T11, 40 cm. from the upper incisors; a tumor 10 cm. proximal to the esophagogastric junction stricture is seen endoscopically at 30 cm.


11. Which of the following statements about pathology encountered at esophagoscopy is/are correct?
A. Reflux esophagitis should be graded as mild, moderate, or severe, to promote consistency among different observers.
B. An esophageal reflux stricture with a 2-mm. lumen is not dilatable and is best treated with resection.
C. A newly diagnosed radiographic distal esophageal stricture warrants dilation and antireflux medical therapy.
D. In patients with Barrett's mucosa, the squamocolumnar epithelial junction occurs 3 cm. or more proximal to the anatomic esophagogastric junction.
E. After fasting at least 12 hours, a patient with megaesophagus of achalasia can safely undergo flexible fiberoptic esophagoscopy.
Answer: D

DISCUSSION: The traditional subjective grading of reflux esophagitis as mild, moderate, or severe has inherent wide variations in meaning among observers. Consistent use of standardized grading systems for endoscopic reflux esophagitis (e.g., that of Belsey or Savary) provides a more objective description of the changes seen and allows more meaningful evaluation of patients at different times and by different observers. The size of the lumen does not predict whether or not a reflux stricture is dilatable. Even a tight 2-mm. lumen can be traversed with a guidewire over which Savary dilators can be used to achieve an acceptable lumen size. Every newly diagnosed esophageal stricture warrants esophagoscopy with brushings and biopsies of the stricture (to exclude carcinoma) and an assessment of its “dilatability.” Antireflux medical therapy is not justified until carcinoma has been ruled out. Because the squamocolumnar epithelial junction may normally be found within 2 to 3 cm. of the anatomic esophagogastric junction, the diagnosis of Barrett's mucosa requires identification of the columnar epithelium at least 3 mm. proximal to the junction of the tubular esophagus and the stomach. In advanced achalasia with megaesophagus, the dilated esophagus may have a capacity of 1 to 2 liters, and simply fasting overnight does not ensure that the esophagus is empty of food and drink consumed the day before. Life-threatening massive regurgitation and aspiration may occur as the endoscope is being introduced unless an effort is made to evacuate the esophagus first by means of a nasogastric tube.


12. Which of the following statements about the diagnosis and treatment of esophageal leiomyomas is/are correct?
A. The majority are diagnosed after they cause dysphagia and chest pain.
B. Biopsy is indicated at the time of esophagoscopy, to rule out carcinoma.
C. Full-thickness elliptical excision of the esophageal wall is the preferred surgical approach.
D. Endoscopic ultrasonography is a reliable means of following leiomyomas conservatively.
E. Recurrence of resected leiomyomas is minimized by wide local excision.
Answer: D

DISCUSSION: Most esophageal leiomyomas are asymptomatic when discovered incidentally on a barium esophagogram or upper gastrointestinal tract series. When suspected on the basis of its radiographic appearance, biopsy of the mass should not be performed at the time of esophagoscopy, so that subsequent extramucosal resection will not be complicated by scarring at the biopsy site. The preferred surgical approach is submucosal enucleation of the mass, not full-thickness excision. Leiomyomas have a characteristic hypoechogenic homogeneous appearance on esophageal ultrasonography that allows a noninvasive diagnosis and means of surveillance. Submucosal enucleation of leiomyomas, without wide local excision, provides excellent long-term results with virtually no local recurrence rate.


13. Which of the following statements regarding the pathology of esophageal carcinoma is/are correct?
A. Worldwide, adenocarcinoma is the most common esophageal malignancy.
B. Squamous cell carcinoma is most common in the distal esophagus, whereas adenocarcinoma predominates in the middle third.
C. Patients with Barrett's metaplasia are 40 times more likely than the general population to develop adenocarcinoma.
D. Metastases from esophageal carcinoma are characteristically localized to regional mediastinal lymph nodes adjacent to the tumor.
E. Achalasia, radiation esophagitis, caustic esophageal stricture, Barrett's mucosa, and Plummer-Vinson syndrome are all premalignant esophageal lesions that predispose to the development of squamous cell carcinoma.
Answer: C

DISCUSSION: Histologically, 95% of esophageal cancers worldwide are squamous cell carcinomas, but the incidence of adenocarcinoma is increasing dramatically in the United States and Europe. Squamous cell carcinoma predominates in the upper and middle thirds of the esophagus, whereas adenocarcinoma is the most frequent distal esophageal cancer. A columnar lined lower esophagus (Barrett's metaplasia) is associated with an incidence of adenocarcinoma approximately 40 times greater than that of the general population. Esophageal cancer is a biologically aggressive tumor that characteristically metastasizes widely to regional and distant lymph nodes as well as to liver and lungs. Recognized premalignant esophageal lesions include achalasia, radiation esophagitis, caustic stricture, Plummer-Vinson syndrome, leukoplakia, esophageal diverticula, and Barrett's metaplasia. All but Barrett's metaplasia are associated with the development of squamous cell carcinoma.


14. Which of the following statements about the surgical treatment of esophageal carcinoma is/are correct?
A. The finding of severe dysphagia in association with Barrett's mucosa is an indication for an antireflux operation to prevent subsequent development of carcinoma.
B. Long-term survival is improved by radical en bloc resection of the esophagus with its contained tumor, adjacent mediastinal tissues, and regional lymph nodes.
C. The morbidity and mortality rates for cervical esophagogastric anastomotic leak are substantially less than those associated with intrathoracic esophagogastric anastomotic leak.
D. The leading complications of transthoracic esophagectomy and intrathoracic esophagogastric anastomosis are bleeding and wound infection.
E. Transhiatal esophagectomy without thoracotomy achieves better long-term survival than transthoracic esophagectomy.
Answer: C

DISCUSSION: Severe dysplasia in Barrett's mucosa is indicative of carcinoma in situ and is an indication for resectional therapy, not an antireflux operation. In the majority of patients, local tumor invasion or distant metastases preclude cure when esophageal carcinoma is diagnosed, and attempts to improve survival with a more radical local operation performed in the face of systemic disease have been disappointingly futile. A cervical esophagogastric anastomotic leak causes a relatively minor cervical salivary fistula that heals in 7 to 10 days in 95% of patients. In contrast, an intrathoracic esophagogastric anastomotic leak results in mediastinitis, which is fatal in 50%. The leading complications of transthoracic esophagectomy and an intrathoracic esophagogastric anastomosis are respiratory insufficiency (from combined thoracic and abdominal incisions) and anastomotic leak resulting in mediastinitis and sepsis. Both complications are minimized by transhiatal esophagectomy without thoracotomy plus cervical esophagogastric anastomosis. No single operative approach to the treatment of esophageal cancer has proved superior to others in terms of long-term survival. The biologic behavior of the tumor (its stage and aggressiveness)—not the number of lymph nodes resected with the tumor—determines survival.


15. The best management for a 48-hour-old distal esophageal perforation is:
A. Antibiotics and drainage.
B. Division of the esophagus and exclusion of the perforation.
C. Primary repair with buttressing.
D. Resection with cervical esophagostomy, gastrostomy, and jejunostomy.
E. T-tube fistula and drainage.
Answer: C

DISCUSSION: When the esophagus is repaired primarily and covered by well-vascularized autologous tissue, the rates of fistula and death are significantly less than those observed for patients who receive simple repair without any protection. Primary repair with buttressing is the first choice for treatment. Resection is reserved for esophageal perforations with extensive damage to the esophageal wall or with advanced mediastinal infection and sepsis. Exclusion of the perforated esophagus and T-tube drainage of a perforation are alternative approaches that cannot be considered for primary treatment. Antibiotics and drainage as the sole treatment is reserved for a very small, selected population of patients with well-contained esophageal perforation.


16. A 50-year-old patient develops sudden left lower chest pain and epigastric pain after vomiting. The patient shows diaphoresis, breath sounds are decreased on the left, and there is abdominal guarding. The most appropriate diagnostic test is:
A. Aortography.
B. Esophagoscopy.
C. Electrocardiogram.
D. Film of the chest.
E. White blood count.
Answer: D

DISCUSSION: The history of pain after vomiting efforts suggests esophageal rupture. Pain is often described as excruciating and frequently masquerades as a dissecting aneurysm, perforated ulcer, or myocardial infarction. Decreased breath sounds suggest the possibility of hydropneumothorax. The diagnostic procedure is a chest film. More than 90% of patients with an esophageal perforation show abnormalities suggestive of perforation. The findings are influenced by the interval between perforation and the examination, by the site of the perforation, and by the integrity of the mediastinal pleura. Esophagoscopy is not indicated.


17. The following statements about the influence of diet and lifestyle on lower esophageal sphincter (LES) function are true except one. Identify the incorrect statement.
A. A high-protein diet increases LES pressure.
B. A fat meal results in sustained decrease in LES pressure.
C. Chocolate ingestion causes a decrease in LES pressure.
D. Peppermint produces a transient decrease in LES values.
E. Cigarette smoking produces no significant changes in LES pressures.
Answer: E

DISCUSSION: There is a dramatic decrease in LES pressure following the ingestion of fat. Chocolate has the same effect on LES resting pressures. Peppermint was shown to produce transient decreases in LES pressures of 20 to 30 seconds, which occur approximately 10 minutes after ingestion. The ingestion of carbohydrates produces no significant change in sphincter pressures, but a high-protein meal increases LES pressure. Cigarette smoking has also been shown to produce significant decreases in LES pressure that persist throughout the duration of active smoking.


18. When a stricture is present in association with gastroesophageal reflux, each of the following is an acceptable repair for reflux control except one. Identify the poorest repair.
A. Intrathoracic total fundoplication.
B. Lengthening gastroplasty with total fundoplication.
C. Total fundoplication.
D. Lengthening gastroplasty with partial fundoplication.
E. Partial fundoplication.
Answer: E

DISCUSSION: When a stricture is present, periesophagitis and shortening limit the chances of obtaining a sufficient length of intra-abdominal esophagus. Even extensive mobilization of the esophagus to the aortic arch and freeing of the esophagogastric junction does not afford a comfortable 4 to 5 cm. of esophagus under the diaphragm. A tension-free repair is not possible in such circumstances. Partial fundoplication at this stage of the disease is followed by a 45% failure rate. Excellent results have been reported using a total fundoplication following dilatation of the stricture, and intrathoracic fundoplication provided good results. The lengthening gastroplasty with a partial fundoplication or with total fundoplication shows satisfactory control of reflux in a majority of patients.


19. When assessing gastroesophageal reflux disease by manometry each of the following statements is correct except one. Identify the incorrect one.
A. Absent or extremely low LES pressures have predictive value in identifying more severe reflux.
B. Peristaltic dysfunction increases with increasing severity of esophagitis.
C. With established reflux disease the UES is hypertensive.
D. Esophageal functional changes are worst in patients with a circumferential columnar-lined esophagus.
E. Absence of peristalsis may be associated with more severe forms of reflux disease.
Answer: C

DISCUSSION: In reflux disease when LES pressure is below 10 mm. Hg, manometry is too imprecise to identify a potential for significant reflux. If the pressure is less than 6 mm. Hg, this shows a reasonable high specificity as compared with abnormal reflux on pH testing. When LES pressure is extremely low or nonexistent, this identifies a more severe degree of reflux and a poorer prognosis for long-term medical therapy. In the esophageal body, active reflux esophagitis causes altered function. Failed peristalsis increases, and the contractions become weaker. Patients with a columnar-lined esophagus have the worst functional abnormalities. Although distention or acid perfusion in the proximal esophagus can produce a significant increase in UES resting pressure, there is at present no solid evidence relating UES resting pressures to active reflux disease.


20. The presence of a nonmalignant mid- or upper esophageal stricture always indicates the presence of:
A. Alkaline reflux esophagitis.
B. Barrett's esophagus.
C. Idiopathic reflux disease.
D. Mediastinal fibrosis.
E. Scleroderma.
Answer: B

DISCUSSION: A stricture at or above the aortic arch is almost certainly situated above an esophagus lined at its lower end with columnar epithelium. Barrett's esophagus is suggested radiographically when local esophagitis, ulcer, or stricture is at the limits of a normal-looking segment of esophagus under the stricture but above a herniated stomach. The columnar-lined esophagus is not always associated with a high stricture; however Messiaen and Halpert documented strictures in 80% of their Barrett's patients. These high strictures, when seen with alkaline reflux esophagitis, with idiopathic reflux disease, or in association with scleroderma, always suggest the presence of a columnar-lined esophagus. Mediastinal fibrosis is a rare condition that can cause multilevel strictures on the esophagus.


21. Which of the following is most reliable for confirming the occurrence of a significant esophageal caustic injury?
A. History of the event.
B. Physical examination of the patient.
C. Barium esophagraphy.
D. Endoscopy.
Answer: D

DISCUSSION: In the absence of physical or radiographic evidence of upper airway obstruction or esophagogastric perforation, the presence of a significant caustic injury can be defined reliably only by direct visualization at the time of endoscopy. Although the history may shed light on the possibility of a burn and its severity, all too often the event goes unwitnessed or the type and amount of ingested substance are not known with certainty. The identification of oropharyngeal burns clearly indicates the need for endoscopy, but as many as 70% of patients with such lesions escape associated esophageal injury. Conversely, 10% to 30% of patients with no external evidence of burns have subsequently been confirmed by esophagoscopy to have sustained damage. In the absence of an identifiable perforation, a barium esophagogram can rarely be considered unequivocally diagnostic of acute injury, though such an injury may be suggested when the esophagus appears atonic and dilated, rigid and persistently narrowed, or excessively irritable. Because of the importance of early confirmation of the presence or absence of a significant esophageal burn as a guide to formulating appropriate treatment, esophagoscopy should be performed expeditiously as soon as sufficient time has elapsed to allow gastric emptying and stabilization of the patient, preferably within the first 12 to 48 hours after ingestion.


22. Indications for surgical reconstruction of the esophagus include which of the following?
A. Continuing requirement for frequent dilation of an extensive esophageal stricture for a minimum of 2 years.
B. Failure or refusal of the patient to comply with a treatment regimen of regular dilation.
C. Development of a fistula between the esophagus and tracheobronchial tree.
D. Iatrogenic perforation of the esophagus during attempted dilation.
Answer: BCD

DISCUSSION: Development of a tracheobronchial fistula almost always necessitates some form of esophageal reconstruction because of the extensive damage usually associated with it. Failure of the patient to cooperate effectively condemns to failure any attempt at restoring esophageal patency by bougienage, because sporadic attempts at dilation do not allow progressive lumen enlargement but, rather, invite additional injury because of the necessity for repeated instrumentation of a recurrent, tight stricture. The need for repeated dilation of extensive or multiple strictures over a period exceeding 6 months should prompt surgical reconstruction of the esophagus, especially in young children, for whom the psychological and physical hazards are intensified by prolonged treatment. In such circumstances, a 2-year period of attempted bougienage is excessive. Although iatrogenic perforation often signals the need for esophageal reconstruction, this misadventure should not be considered an absolute indication but should be assessed in relation to (1) the extent and complexity of the stricture, (2) the potential for eventually achieving successful bougienage, and (3) the severity of complications caused by the secondary injury.


23. First-line therapy for routine peptic duodenal ulcer disease includes:
A. Vagotomy and antrectomy.
B. Upper endoscopy and biopsy to rule out tumor.
C. Evaluation for Helicobacter pylori.
D. Serum gastrin determination.
E. Cream or milk-based “Sippy” diet.
Answer: C

DISCUSSION: Vagotomy and antrectomy is the definitive surgical therapy for peptic ulcer disease but should be applied only for complications of the disease or after refractory disease has been documented. Biopsy of routine peptic duodenal ulcer is not indicated to rule out malignancy except in special circumstances, such as an endoscopic appearance typical of malignancy. H. pylori is found in a large percentage of peptic ulcer patients, and treatment alters the rate of recurrence if therapy is directed toward reduction of H. pylori in addition to acid. Measurement of serum gastrin is recommended for patients with resistant or persistent peptic ulcer disease for patients undergoing surgery for peptic ulcer disease. The Sippy diet has not been recommended clinically for years. Formerly it was recommended as a bland diet that would not exacerbate peptic ulcer disease. It is now known that these diets are heavily calcium-laden and probably exacerbate peptic ulcer disease.


24. Appropriate management of severe vomiting associated with gastric outlet obstruction from peptic ulcer disease includes all of the following except:
A. Nasogastric suction.
B. Intravenous hydration.
C. Nutritional assessment; upper endoscopy to rule out malignancy.
D. Intravenous H 2 antagonist.
E. Oral antacid therapy.
Answer: E

DISCUSSION: All patients should undergo nasogastric suction, rehydration, and control of acid secretion. This control of acid secretion requires an H 2 antagonist since oral antacids are often inadequate to neutralize the large volume of acid often present in the obstructed stomach. Patients with a long history of obstruction are often nutritionally compromised and need careful nutritional assessment before operative planning.


25. All of the following are complications of peptic ulcer surgery except:
A. Duodenal stump blowout.
B. Dumping.
C. Diarrhea.
D. Delayed gastric emptying.
E. Steatorrhea.
Answer: E

DISCUSSION: Duodenal stump blowout occurs after Billroth II operations, where back-pressure in the duodenal stump results in breakdown of this stump closure, leading to abdominal sepsis. Dumping syndrome and postvagotomy diarrhea are complications of peptic ulcer surgery. They represent two different syndromes, both of which are predominately related to the vagotomy portion of the operation. Delayed gastric emptying occurs frequently after peptic ulcer surgery, for a variety of reasons, but it is most common after elective peptic ulcer surgery for gastric outlet obstruction. Steatorrhea is not necessarily related to peptic ulcer surgery but is a complication of pancreatic insufficiency.


26. The presentation of Zollinger-Ellison syndrome includes all of the following except:
A. Hyperparathyroidism in patients with multiple endocrine neoplasia type 1 (MEN 1) syndrome.
B. Diarrhea.
C. Migratory rash.
D. Jejunal ulcers.
E. Duodenal ulcers.
Answer: C

DISCUSSION: Zollinger-Ellison syndrome occurs in two settings: sporadically and in association with MEN 1 syndrome. MEN 1 syndrome includes parathyroid adenomas, and the initial presentation is often related to this parathyroid disease. Diarrhea is a common presentation for Zollinger-Ellison syndrome, since hyperacidity can result in diarrhea due to the volume of acid secreted or from a steatorrhea-type diarrhea when the high levels of acids inactivate the pancreatic enzymes. Migratory rash is commonly associated with glucagonoma but not with Zollinger-Ellison syndrome. Both jejunal and duodenal ulcers can be found with Zollinger-Ellison syndrome.


27. All are true about the dumping syndrome except:
A. Symptoms can be controlled with a somatostatin analog.
B. Diarrhea is always part of the dumping syndrome.
C. Flushing and tachycardia are common features of the syndrome.
D. Separating solids and liquids in the patient's oral intake alleviates some of the symptoms of the syndrome.
E. Early postoperative dumping after vagotomy often resolves spontaneously.
Answer: B

DISCUSSION: The somatostatin analog octreotide has been used to control the dumping syndrome and is currently the only known medical therapy for this disease. Other therapies include dietary measures such as six small meals a day and separation of solids and liquids. Postvagotomy diarrhea is a secondary complication of vagotomy and is not strictly associated with the dumping syndrome itself. The vast majority of patients with dumping syndrome experience spontaneous resolution of their symptoms without intervention in the postoperative period.


28. In patients with bleeding duodenal ulcers, the endoscopic finding associated with the highest incidence of rebleeding is:
A. Visible vessel.
B. Cherry-red spot.
C. Clean ulcer bed.
D. Duodenitis.
E. Shallow, 3-mm. ulcer.
Answer: A

DISCUSSION: A visible vessel in an ulcer bed is associated with a 50% chance of rebleeding and, other than an actively bleeding vessel, is the worst endoscopic prognostic indicator for rebleeding. Cherry-red spot, adherent clot, and clean small ulcers all are associated with a lower incidence of rebleeding.


29. All of the following are contraindications for highly selective vagotomy except:
A. Intractable duodenal ulcer disease.
B. Peptic ulcer disease causing gastric outlet obstruction.
C. Fundic peptic ulceration.
D. Cigarette chain smoking.
E. Perforated peptic ulcer disease with more than 24 hours' soilage.
Answer: A

DISCUSSION: Intractable peptic ulcer symptoms are a classic indication for highly selective vagotomy. Patients with gastric outlet obstruction often do poorly with highly selective vagotomy and develop recurrent ulceration. Highly selective vagotomy is not indicated for gastric ulceration. Heavy chain smokers often get recurrent peptic ulceration after highly selective vagotomy; therefore, vagotomy and antrectomy is indicated for them. Patients who experience long periods of perforation before exploratory laparotomy should receive either patch plus vagotomy or pyloroplasty or patch of the ulcer alone. Extensive operations, such as highly selective vagotomy, are usually not indicated in this acute setting.


30. All the following are true of omeprazole except:
A. It is the only drug available that has the potential to achieve pharmacologically induced achlorhydria.
B. It works by blocking the hydrogen-potassium ATPase in the parietal cell.
C. It is parietal cell specific.
D. It has a short half-life (about 90 minutes) when taken orally.
E. It has been associated with gastric neoplasm in a rat model.
Answer: D

DISCUSSION: Omeprazole and drugs in this category are the only drugs that can produce achlorhydria. All other antiacid drugs reduce acid secretion without producing achlorhydria. Omeprazole inhibits acid at the final common pathway by blocking the hydrogen-potassium ATPase in parietal cells. It is gastric parietal cell specific and has a very long half-life, allowing once daily dosing when given orally. When it was given to rats in pharmacologic doses the gastric mucosa formed carcinoid-type tumors. This problem has not been identified in humans.


31. All of the following statements about gastrin-releasing peptide (GRP) are true except:
A. In species other than man and dog GRP is commonly referred to as bombesin.
B. GRP serves as a neurotransmitter.
C. GRP inhibits pancreatic secretion when given intravenously.
D. GRP stimulates gastric acid secretion when given intravenously.
E. GRP is released in response to cholinergic stimulation of the parietal cells to stimulate release of gastrin.
Answer: C

DISCUSSION: Gastrin-releasing peptide and bombesin are homologous peptides of different amino acid lengths. GRP functions as a neurotransmitter at the cholinergic nerve ending on the parietal cell and releases gastrin after cholinergic stimulation. It functions to increase gastric acid secretion and also pharmacologically increases pancreatic secretion.


32. Cholecystokinin (CCK) is believed to function in all of the following processes except:
A. It physiologically delays gastric emptying.
B. It appears to have a role in satiety regulation.
C. It contracts the gallbladder.
D. It stimulates pancreatic secretion.
E. It is important in the control of the anal sphincter.
Answer: E

DISCUSSION: CCK has a physiologic role in the regulation of gastric emptying, eating behavior, gallbladder contraction, and pancreatic secretion. There is experimental evidence that it may serve as a neurotransmitter in the function of the lower esophageal sphincter. It probably also has a role in augmenting the release of insulin after a meal. It has no known role in the function of the anal sphincter.


33. All of the following measures have been recommended for control of acid secretion in patients with Zollinger-Ellison syndrome except:
A. Antrectomy.
B. Highly selective vagotomy.
C. Total gastrectomy.
D. Vagotomy and pyloroplasty.
E. Medical therapy with Prilosec (omeprazole).
Answer: A

DISCUSSION: Patients with MEN 1 syndrome or sporadic-metastatic Zollinger-Ellison syndrome should be palliated with omeprazole to control their acid secretion. Patients who undergo exploration may have a variety of operations to control their ulcer diathesis, including total gastrectomy or various vagotomy-type operations. Antrectomy alone is not indicated, since the gastrin that is contributing to the production of acid is not coming from the antrum but coming from the tumor.


34. All of the following contribute to peptic ulcer disease except:
A. Cigarette smoking.
B. Nonsteroidal anti-inflammatory drugs.
C. Helicobacter pylori.
D. Gastrinoma.
E. Spicy foods.
Answer: E

DISCUSSION: Cigarettes and nonsteroidal anti-inflammatory drugs are common contributors to peptic ulceration. H. pylori is found in most patients with peptic ulceration, and eradication of this bacterium decreases the recurrence rate for peptic ulcer disease. Gastrinoma results in much acid secretion and commonly presents with peptic ulcer disease. Dietary factors such as spicy foods have little or no effect on postprandial acid secretion and do not contribute to peptic ulceration.


35. Which of the following statements about gastric polyps is/are true?
A. Like their colonic counterparts, gastric epithelial polyps are common tumors.
B. They are analogous to colorectal polyps in natural history.
C. Endoscopy can uniformly predict the histology of a polyp based on location and appearance.
D. In a given patient, multiple polyps are generally of a single histologic type.
E. Gastric adenomatous polyps greater than 2 cm. in diameter should be excised because of the risk of malignant transformation.
Answer: DE

DISCUSSION: As early as 1895 Hauser reported an association between familial adenomatous polyposis of the colon and multiple gastric polyps. This early association may have given rise not only to the confusing nomenclature of gastric polyps but also to the mistaken notion that they are analogous to colorectal polyps in microscopic appearance and natural history. Unlike colonic polyps, gastric epithelial polyps are very uncommon tumors (prevalence 0.4% to 0.8%). Their histologic appearance cannot be predicted on the basis of location in the stomach, although the endoscopic literature is beginning to define predictive algorithms based on location and ultrasound. Multiple polyps are almost always of a single histologic type. Gastric adenomatous polyps have long been associated with adenocarcinoma. This association is directly related to the size of the polyps. Up to 24% of polyps 2 cm. or greater in diameter are associated with adenocarcinoma. In contrast, only 4% of polyps with a diameter less than 2 cm. are associated with carcinoma. The risk, if any, of carcinoma in patients with hyperplastic polyps appears to be associated with the atrophic gastritis that frequently accompanies them rather than with the polyps themselves.


36. Which of the following statements about gastric leiomyomas is/are true?
A. They are the most common type of gastric tumor of the stomach at autopsy.
B. The leiomyoblastoma cell type reflects malignant transformation of gastric leiomyomas.
C. A conservative surgical approach is indicated for their resection since regional lymphadenectomy has not been proved reliable even when they turn out to be malignant.
D. Severe hemorrhage may occur from deep ulcerations overlying the intramural tumor.
Answer: ACD

DISCUSSION: Approximately 40% of benign tumors of the stomach are leiomyomas derived from the smooth muscle of the stomach or its associated blood vessels. Because it is rare for gastric leiomyomas smaller than 3 cm. in diameter to be symptomatic, considerably fewer than 2% of gastric neoplasms resected surgically are of smooth muscle origin. Gastric leiomyomas may be smooth or lobulated, but in time a central ulceration occurs in the mucosal bulge of the tumor in approximately half of submucosal leiomyomas. Ulceration may be present in smaller tumors but absent in very large tumors. Overlying central mucosal ulceration, which may penetrate deeply into the tumor, results in hematemesis, melena, or anemia and draws attention to the tumor. Bleeding from the tumor may be massive and/or intermittent.
Gastric leiomyomas are not encapsulated, even though on section they appear to be well-circumscribed. Microscopically, the tumor cells at the margin may intermingle with cells of the surrounding gastric wall. Along with the presence of occasional large cells with hyperchromatic nuclei, this has led to confusion in distinguishing benign tumors from malignant ones. Stout described a reasonably distinct variety of gastric smooth muscle tumor that he called leiomyoblastoma (bizarre smooth muscle tumor). They were characterized histologically by polyhedral smooth muscle cells with central nuclei and abundant cytoplasm rather than elongated cells. A clear zone that surrounds the central nucleus may be an artifact of fixation. Leiomyoblastoma may be benign or malignant. Carney has described a syndrome characterized by the triad of multiple malignant leiomyoblastoma, pulmonary chondroma, and functioning extra-adrenal paraganglioma.
The principle of surgical treatment of smooth muscle tumors is local excision with a 2- to 3-cm. margin of surrounding gastric wall. In view of the difficulty in distinguishing between the benign and malignant variants, enucleation is not an appropriate method of treatment. Regional lymphadenectomy is not of proven value, even if malignancy is strongly suspected and is not consistent with the known property of these tumors to spread by the hematogenous route.


37. The sine qua non of the histologic diagnosis of a gastric pseudolymphoma is:
A. Extragastric extension of the gastric lesion.
B. Nodal involvement beyond the immediate stomach.
C. A germinal center in the gastric lesion.
D. Extension into esophagus and duodenum.
E. Unresponsive to conservative gastric resection.
Answer: C

DISCUSSION: Pseudolymphoma represents approximately 10% of all gastric lymphomas. These are benign conditions involving mostly the mucosa without evidence of nodal disease and without extragastric extension. The sine qua non for the diagnosis is a germinal center within the gastric lesion. These are premalignant lesions but can be cured completely with conservative resection.


38. All of the following statements about surgical management of gastric lymphomas are true except:
A. Stage I gastric lymphomas (small lesions confined to the stomach wall) can be cured completely with surgical therapy alone.
B. Extensive gastric lymphomas that initially are treated with radiation and/or chemotherapy occasionally perforate during treatment and require secondary resection.
C. Patients explored with a presumptive diagnosis of gastric lymphoma should undergo an attempt at curative resection when this is safe and feasible.
D. Without a preoperative diagnosis resection for gastric mass should not be attempted unless lymphoma can be excluded.
E. Appropriate staging for primary gastric lymphoma includes bone marrow biopsy.
Answer: D

DISCUSSION: Operation alone is adequate treatment for very early-stage lymphoma, although chemotherapy is commonly added. For more advanced disease, particularly stages III and IV, preoperative radiation chemotherapy is often indicated, even though some of these patients suffer perforation during therapy and require emergent resection. Patients who undergo exploration for gastric mass without a preoperative diagnosis can safely be resected with potential for cure even if the diagnosis includes gastric lymphoma.


39. Which of the following risk factors have been shown to increase significantly the incidence of gastrointestinal bleeding from stress gastritis in intensive care unit (ICU) patients?
A. Glucocorticoid administration.
B. Respiratory failure.
C. Coagulopathy.
D. Organ transplantation.
E. Jaundice.
Answer: BC

DISCUSSION: Prophylactic measures such as H 2-receptor antagonists and antacid titration effectively reduce the incidence of gastrointestinal bleeding; however, prophylaxis against stress gastritis is expensive and may have adverse effects. Therefore, it should be used selectively in patients with high risk factors. In a prospective multicenter study in which 10 potential risk factors were evaluated for stress gastritis bleeding in ICU patients, respiratory failure and coagulopathy are two independent risk factors for clinically significant bleeding. Therefore, a strong recommendation for prophylaxis of stress gastritis can be made for ICU patients who have either respiratory failure or coagulopathy.


40. Which of the following measures are effective in preventing stress gastritis bleeding in critically ill patients?
A. Improving systemic circulation by correcting any shocklike state resulting from blood loss or sepsis.
B. Correcting systemic acid-base abnormality.
C. Maintaining adequate nutrition.
D. Reducing intragastric acidity by either antacid titration or H 2 antagonists.
Answer: ABCD

DISCUSSION: Despite the lack of documentation, a strong impression exists among clinicians and clinical investigators that the incidence and prevalence of stress gastritis have decreased significantly during the past decade, perhaps owing to improved general care for critically ill patients. The improvement in general care of these critically ill patients includes vigorous efforts to correct any shocklike state secondary to blood loss or sepsis, better ventilatory support, and maintenance of adequate nutrition. These prophylactic measures enhance the ability of the gastric mucosa to protect itself against acid injury. In addition, several prospective, randomized studies have shown that antacid titration and/or H 2-receptor antagonists are effective in preventing gastrointestinal bleeding in these patients.


41. Which of the following have been used successfully to treat patients with vascular compression of the duodenum?
A. Subtotal gastrectomy and Roux-en-Y gastrojejunostomy.
B. Total parenteral nutrition.
C. Division of the ligament of Treitz and duodenal mobilization.
D. Percutaneous endoscopic gastrostomy.
E. Duodenojejunostomy.
Answer: BCE

DISCUSSION: Vascular compression of the duodenum is best treated initially with supportive care. Of paramount importance is supplying adequate nutrition, since most patients have significant weight loss with this syndrome. This can best be done with a nasojejunal feeding tube placed past the ligament of Treitz (and the obstructed area). Gastrostomy alone does not provide unobstructed enteral access. Parenteral nutrition may be used successfully when enteral access cannot be established. When operative therapy is needed, duodenojejunostomy has been the most common and successful operation and is the treatment of choice for adults. In the pediatric population, division of the ligament of Treitz and duodenal mobilization has also proved successful. Gastrojejunostomy has been used, but with a lower overall success rate. Distal gastrectomy usually worsens duodenal obstruction by preventing duodenogastric reflux.


42. Which of the following statements about the anatomic basis for the syndrome of vascular compression of the duodenum are true?
A. The duodenum is obstructed in its distal third as it crosses over the lumbar vertebral column.
B. Structures crossing beneath the superior mesenteric artery include the duodenum, the uncinate process of the pancreas, and the left renal vein.
C. Hyperextension of the back allows the angle of origin of the superior mesenteric artery to widen, lessening the obstruction of the duodenum.
D. Patients are at significant risk for vascular compression of the duodenum if the angle between the takeoff of the superior mesenteric artery and the aorta is less than 45 degrees.
E. Arteriographic studies show a typical area of extrinsic compression and narrowing of the arterial lumen due to duodenal pressure.
Answer: AB

DISCUSSION: The superior mesenteric artery originates behind the neck of the pancreas at the level of the first lumbar vertebra. It arises from the aorta at an acute angle, usually about 37 degrees in normal patients, through which passes the left renal vein, the uncinate process of the pancreas, and the distal third of the duodenum. The duodenum crosses the lumbar spine from right to left and passes upward. It is at this point of passage of the duodenum upward and over the spine that the obstruction occurs. Arteriographic studies show that the aortomesenteric angle in patients with the syndrome is only about 8 degrees. There is no narrowing of the superior mesenteric artery or disturbance of arterial flow, but the area of duodenal obstruction corresponds to the compression of the bowel by the artery. The duodenal compression may often be relieved by assuming the knee-chest, the left lateral, or even the prone position. Increasing lumbar lordosis, as with hyperextension of the back, exacerbates the problem.


43. Numerous epidemiologic associations have been made between (1) environmental and dietary factors and (2) the incidence of gastric cancer, including all except:
A. Dietary nitrites.
B. Dietary salt.
C. Helicobacter pylori infection.
D. Dietary ascorbic acid.
Answer: D

DISCUSSION: Numerous epidemiologic studies support the role of certain foods in the development of gastric cancer. Salt, which can act as a gastric irritant, and nitrates and nitrites, which can be converted to the active carcinogens N-nitrosamines, are implicated in the development of gastric cancer. H. pylori infection is associated with atrophic gastritis, a known precursor to gastric cancer. Important studies of large populations indicate that the majority of patients with gastric cancer are H. pylori positive. The bacteria produce toxins such as ammonia and acetaldehyde, which could lead to chronic inflammation and epithelial damage. Dietary ascorbic acid has been associated with overall improvements in diet and is not associated with the development of gastric cancer.


44. All of the following benign conditions are associated with increased rates of gastric cancer except:
A. Pernicious anemia.
B. Multiple endocrine neoplasia type I (MEN 1).
C. Adenomatous polyps.
D. Chronic atrophic gastritis.
Answer: B

DISCUSSION: Adenomatous polyps are unusual but carry the distinct potential for malignancy. They occur most often between the fifth and seventh decades of life. The adenocarcinoma sequence in gastric polyps is thought to be analogous to that of colonic polyps. An adenomatous polyp is a marker for increased risk of carcinoma in the remaining stomach. Both pernicious anemia and chronic atrophic gastritis are associated with gastric cancer. Many of these patients develop chronic achlorhydria, a condition also associated with an increased risk of cancer. Neither multiple MEN 1 nor MEN 2, is associated with gastric cancer.


45. Which of the following statements concerning the pathology of gastric cancer is true?
A. Distal gastric cancers are becoming more common.
B. Intestinal-type gastric tumors resemble colon carcinomas and have a better prognosis than diffuse type.
C. Early gastric cancers are confined to the mucosa and are lymph node negative.
D. Broders' histologic grading system correlates well with survival: patients with grade IV tumors have 5-year survival rates around 65%.
Answer: B

DISCUSSION: Distal gastric cancers are decreasing in incidence in several populations. Lesions of the gastroesophageal junction and cardia have increased in incidence over the past two decades. Early gastric cancers are confined to the mucosa and submucosa of the stomach. Six to 10% of these early lesions are lymph-node positive. The survival rates from early gastric cancer is related to node positivity, just as in advanced gastric cancer. Broders' histologic grades do correlate well with survival. Grade I and IV tumors are associated with a 65% and 11% 5-year survival, respectively. The Lauren classification system is divided into intestinal and diffuse-type tumors. The intestinal-type tumor is more analogous to colon carcinoma and has a better prognosis than the diffuse type.


46. An 80% distal gastrectomy is performed for a 6-cm. antral cancer with extension to the muscularis propria and three positive lymph nodes less than 3 cm. from the tumor. The stage of this tumor was:
A. Stage I.
B. Stage II.
C. Stage III A.
D. Stage III B.
Answer: B

DISCUSSION: The American Joint Committee on Cancer Staging system depends on primary tumor, lymph node involvement, and distant metastasis. The tumor described is a T2N1M0 tumor, which categorizes it as stage II.


47. Which of the following statements about the surgical treatment of gastric cancer is false?
A. Patients with tumors of the middle and proximal thirds should undergo total gastrectomy.
B. Adenocarcinoma of the cardia-gastroesophageal junction may require reconstruction in the abdomen, chest, or neck.
C. Palliative resection yields better results than palliative bypass.
D. Japanese patients who undergo gastric resection are, on average, 10 years younger and much leaner than their Western counterparts.
Answer: A

DISCUSSION: Depending on the size and extent of the tumor, cancers of the gastroesophageal junction may extend proximally into the esophagus for a varying distance. Reconstruction may be required in the abdomen, chest, or neck, depending on extension and whether the operation is to be palliative or curative. Palliative resection yields better results than palliative bypass, which is unreliable for relieving obstruction. Japanese patients typically are younger and thinner than their Western counterparts. In addition, they have a higher prevalence (up to 50%) of early gastric cancer. Depending on the size and particular location of the tumor, patients with small middle-third tumors or small lesions of the cardia may undergo subtotal proximal gastrectomy and reconstruction with a gastric tube. If 6-cm. margins can be obtained on either side of the lesion, total gastrectomy is unnecessary and may be associated with a higher risk of morbidity.


48. Which of the following measures of obesity correlates best with mortality?
A. The 1983 Metropolitan Life Insurance Company tables for ideal body weight.
B. Hydroimmersion measurements of body fat composition.
C. Body mass index (BMI).
D. Skinfold thickness.
E. Waist to hip ratios (WHR).
Answer: C

DISCUSSION: The measurement of obesity is still an inexact science. The Metropolitan Life Insurance tables, although widely used, do not distinguish between lean muscle mass and fat. Accordingly, the BMI (weight in kg./height in meters) 2, was developed to place greater emphasis on “fatness.” The measure correlates linearly with mortality tables. Hydroimmersion data are still too sparse to relate to outcome tables. Skinfold thickness and waist-hip ratios have not been shown to have the accuracy or relevance of the BMI.


49. The most effective therapy for morbid obesity, in terms of weight control, is:
A. Intensive dieting with behavior modification.
B. A multidrug protocol with fenfluramine, phenylpropanolamine, and mazindol.
C. A gastric bypass with a 40-ml. pouch, a 10- to 20-cm. Roux-en-Y gastroenterostomy.
D. A gastric bypass with a 15-ml. pouch, a 40- to 60-cm. Roux-en-Y gastroenterostomy.
E. Daily exercise with strong emphasis on utilizing all four limbs.
Answer: D

DISCUSSION: Although the various nonsurgical measures listed in the question have proved effective for obese persons, they work only rarely for those who are morbidly obese. None have proved as effective as gastric bypass with a 15-ml. pouch and a 40- to 60-cm. Roux-en-Y gastroenterostomy.


50. Which of the following statements about intestinal bypass is/are correct?
A. The operation produced weight loss similar to that of the gastric bypass.
B. The operation produced severe metabolic disturbances, including hypocalcemia, increased bile salts and glycine synthesis.
C. Bacterial overgrowth in the bypassed segment led to liver failure.
D. The operation demonstrated that an adult human could survive with 40 to 50 cm. of small intestine.
Answer: ABCD

DISCUSSION: Unfortunately, all of the answers are true. Even though the intestinal bypass proved initially to be an effective procedure to induce weight loss, the side effects proved to be so severe that almost all of the operations had to be reversed or revised to gastric bypass to avert death from liver failure or severe illnesses due to malnutrition.


51. Which of the following is/are contraindications to gastric bypass surgery?
A. Diabetes mellitus.
B. Hypertension.
C. Pickwickian syndrome.
D. Failure to agree to long-term follow-up.
E. Sleep apnea.
Answer: D

DISCUSSION: The gastric bypass represents the best known therapy for diabetes mellitus, hypertension, the Pickwickian syndrome, and sleep apnea. In fact, no other therapy provides such complete control of hyperglycemia and hyperinsulinemia, reversal of hypertension, and total correction of the Pickwickian syndrome and most cases of sleep apnea. The only contraindication to bariatric surgery listed is failure to agree to long-term follow-up. The gastric bypass represents controlled malnutrition and, therefore, vitamin therapy is especially important. If patients are not followed closely, vitamin deficiencies, especially of B 6 and B 12, can develop with serious consequences including a Korsakoff-Wernicke syndrome.


52. A 34-year-old morbidly obese diabetic woman underwent a gastric bypass about 12 hours ago. The operation was technically difficult but finally went well. You are called because she now has a temperature of 99.2؛ F, pulse of 134, and some pain in her incision and her back. She looks well; the incision is clean; and her examination is otherwise negative. A bolus of 500 ml. of dextrose/lactated Ringer's did not change her vital signs, except that her pulse rose to 140 without an increase in urine output. Your next step should be:
A. Another bolus of crystalloids.
B. Posterioanterior and lateral chest films.
C. Obtain white cell count, differential count, and electrolyte values.
D. Call the operating room and warn them that you need to re-explore for a leak.
E. Increase her pain medication.
Answer: D

DISCUSSION: Morbidly obese patients are malnourished and brittle and have little resistance. Serious life-threatening infections may soil the peritoneal cavity without producing any sign except a persistently high pulse rate. The usual tests listed in A, B, and C, should be done, but the most likely explanation for a continued high pulse is soiling in the area of the surgery due to either a leak or contamination and development of sepsis. Because of the unreliability of clinical evaluation, the indications for re-exploration are very liberal, and this approach has saved a number of lives. The risk of such an exploration is small, whereas failure to contain the infection with lavage and drainage may be followed by a surprisingly rapid death.



53. Metabolic complications of subtotal gastrectomy with Billroth I or Billroth II reconstruction include:
A. Hypothyroidism.
B. Anemia.
C. Reactive hypoglycemia.
D. Dumping syndrome.
E. Metabolic bone disease.
Answer: BCDE

DISCUSSION: Anemia develops in as many as 30% of patients within 15 years of surgery. The cause is multifactorial and includes malabsorption of iron, folate, and vitamin B 12. A metabolic bone disease occurs in as many as 33% of patients, is similar to osteomalacia, and is probably a result of malabsorption of calcium and vitamin D. Reactive hypoglycemia occurs with rapid gastric emptying, resulting in increased glucose absorption immediately after a meal. Initially there is hyperglycemia, leading to hyperinsulinemia and subsequent rapid glucose clearance and symptomatic hypoglycemia. Dumping syndrome varies from very mild symptoms to significantly disabling ones. The severe syndrome occurs in fewer than 5% of patients. Small, frequent, dry meals of low osmolality reduces symptoms, and somatostatin analog has been of some clinical use.


54. Which of the following statement(s) concerning the surgical options for an anti-reflux operation is/are true?

a. A patient with normal esophageal length and esophageal body motility is best served by laparoscopic Nissen fundoplication
b. A patient with a low peristaltic amplitude of the distal third of the esophagus is a candidate for an open Nissen fundoplication
c. A Collis gastroplasty is an additional procedure that can be added in patients with extensive esophageal shortening
d. End-stage reflux disease such as an undilatable stricture or Barrett’s esophagus with high grade dysplagia is best managed by a colon interposition
Answer: a, c, d

Patients with normal esophageal length and normal esophageal body motility are best served by a transabdominal Nissen fundoplication. This is now normally done via the laparoscopic route. If the patient is very obese or requires concomitant surgery on the lung or esophageal body, the transthoracic approach is preferable. The presence of a motility disorder alters the operative strategy. If the peristaltic amplitude is low (20 mm Hg) in the distal third of the esophagus, a Nissen fundoplication would create too much resistance and lead to dysphagia. In this situation the Belsey fundoplication is a better choice. Moreover, it allows the surgeon to mobilize the esophagus to a much greater extent than is possible through the abdomen. In addition to extensive mobilization, a Collis gastroplasty can be created to produce an extra 5 cm of “neo-esophagus” around which a Belsey procedure can be added. End-stage reflux disease, for example, when there is an undilatable stricture or after previous unsuccessful anti-reflux operations or when Barrett’s esophagus leads to high grade dysplagia, is best served by esophageal replacement. The most durable substitute is the colon, and the functional results are especially good if the vagus nerves are intact.


55. Factors associated with the development of complications of gastroesophageal reflux disease include:

a. The presence of a defective lower esophageal sphincter
b. Inadequate esophageal clearance
c. The presence of a hiatal hernia
d. The presence of an alkaline component of the reflux material
Answer: a, b, c, d

The status of the lower esophageal sphincter (LES) has emerged as a significant factor in several long-term studies of gastroesophageal reflux disease, and serves as a predictor of a poor response to medical treatment. Barrett’s esophagus is almost always associated with a mechanically defective sphincter. Any defects in esophageal clearance which prolongs the contact time between the refluxate and the mucosa is likely to lead to increased esophageal injury. The presence of a hiatal hernia is also associated with more complications of gastroesophageal reflux disease. Finally, the composition of the reflux material also has an effect on the development of complications. In a clinical situation, complications of gastroesophageal reflux disease are more common when there is an alkaline component to the refluxate. In Barrett’s esophagus, the development of complications such as stricture and ulceration is strongly associated with increased alkaline exposure.


56. Fundamental to understanding disorders of esophageal function is the measurement of the contractility of the esophageal body and sphincters. Which of the following statement(s) is/are true concerning esophageal manometry in the investigation of benign esophageal disease?

a. A defective sphincter is predictive of poor long-term response to medical therapy, but a good response to surgery
b. Esophageal manometry can determine the resting pressure and the overall length of the sphincter but not its abdominal length
c. The LES pressure normally drops to gastric baseline immediately after a swallow before the peristaltic wave reaches the lower esophagus
d. A Vector Volume below the fifth percentile of normal is the most sensitive measure of mechanical deficiency of the LES
e. There is no correlation between defects in LES with the severity of gastroesophageal reflux disease
Answer: a, c, d

Esophageal manometry is an investigative tool in which a catheter containing pressure sensors is inserted into the esophagus and used to measure pressures in esophageal body and sphincters at rest and in response to swallowing. It is indicated in a number of clinical situations including nonobstructive dysphasia, noncardiac chest pain, and the assessment of gastroesophageal reflux disease.
The indications for manometry in patients with suspected gastroesophageal reflux are chiefly to assess the status of the LES and to identify a motility disorder of the body. A defective sphincter is predictive of a poor long-term response to medical therapy, but a good response to surgery. The presence of a motility defect profoundly alters the operative strategy in patients with GERD and should always be excluded by manometry prior to operative therapy. In assessment of the LES, three components are measured: the resting pressure, the overall length of the sphincter, and the abdominal length. A defect in the values for each of these components are determined when the lower limits of normal (fifth percentile) are determined. A defect in one or even two components of the LES may be compensated for by good esophageal body function, but when all three components are defective, excessive esophageal acid exposure is inevitable. All the pressures measured along the length of the sphincter and around its circumference during the pull-through may be treated as vectors having both magnitude and direction and hence integrated into a three-dimensional image, the volume (Vector Volume) of which is a measure of LES resistance. A Vector Volume below the fifth percentile of normal is the most sensitive measure of mechanical deficiency of the LES. The prevalence of a defective LES increases with increasing severity of GERD, being the lowest in patients without evidence of endoscopic injury and highest in patients with stricture or Barrett’s esophagus.


57. Which of the following statement(s) is/are true concerning the diagnosis and management of the patient whose barium esophogram is shown in Figure 18-29?

a. The condition is due to neuronal generation of the myenteric plexus in the lower esophageal sphincter
b. The patient will report symptoms of vomiting of sour or bitter material
c. Despite the impressive radiologic picture, passage of the endoscope through the area of narrowing will likely be possible
d. Manometry and 24 hour pH monitoring should be performed for confirmation of the diagnosis
Answer: c

The x-ray demonstrates moderately advanced achalasia with a dilated esophagus with a narrowed tapering “bird’s beak” appearance of the distal esophagus. Achalasia is the best known primary motility disorder of the esophagus. It is characterized by failure of the esophageal body peristalsis and incomplete relaxation of the LES. It is generally thought to be caused by neuronal degeneration of the myenteric plexus of the esophageal wall, causing aperistalsis, and to loss of activity of the inhibitory neurons in the LES leading to incomplete relaxation. Patients with achalasia all have dysphagia, and most have regurgitation. Careful questioning is needed to distinguish the regurgitation from vomiting. Generally, it occurs during or at the end of a meal, and the material tastes bland rather than sour or bitter. Patients often have to leave the table to regurgitate, and are usually slow eaters.
Endoscopy frequently reveals residual liquid or food in the esophagus. Unlike a stricture, the narrowing of the lower end permits the passage of the endoscope, usually with a characteristic “popping” sensation. In every patient with presumed achalasia, it is very important to view the cardia from below with the endoscope retroflexed, as a small infiltrating gastroesophageal tumor may otherwise be missed. Manometry is required to establish the diagnosis of achalasia. The classic features on stationary manometry are: 1) Elevated LES pressure; 2) Incomplete LES relaxation; 3) Absence of esophageal body peristalsis; and 4) Positive intraesophageal body pressure. Although reports concerning the use of 24 hr pH monitoring appear in the literature, excessive acid exposure is rare.


58. Which of the following statement(s) is/are true concerning other tests available for investigation of esophageal disease?

a. A 24 hour pH monitoring is currently the principal method in making the diagnosis of gastroesophageal reflux disease (GERD)
b. Acid reflux episodes are defined as periods when the esophageal pH is less than 2
c. Twenty-four pH monitoring is only useful in the detection of acid reflux disease
d. The Bernstein test continues to be an important tool in the diagnosis of acid reflux disease
e. Delayed gastric emptying may be an important etiologic factor in patients with GERD
Answer: a, e

The development of 24 hr pH monitoring was a major advance in unraveling the pathophysiology of GERD. It is now the principal method to make the diagnosis of GERD and has effectively replaced all other methods of measuring esophageal acid exposure. It is indicated in any patient with symptoms suggesting GERD, unless the symptoms are trivial, or permanently abolished by a short course of acid suppression therapy. Reflux episodes are defined as periods when the esophageal pH is less than pH 4. Normal (physiologic) reflux occurs in the form of short rapidly cleared postprandial episodes. A few episodes of long duration are more injurious than many brief episodes, even though total acid exposure time may be similar. In addition to the measurement of acid exposure, pH monitoring can also be used to detect excessive alkaline exposure (pH > 7) in the esophagus. The Bernstein test, in which hydrochloric acid is dripped into the esophagus via a nasogastric tube, is sometimes used to determine if a patient’s symptoms are reproduced by acidic exposure. It is basically a measure of esophageal mucosal sensitivity. It has been largely superseded by the use of 24 hr pH monitoring. Gastric emptying is affected by the composition and consistency of the ingested meal. Delayed gastric emptying may be an important etiologic factor in patients with GERD and a normal LES.


59. The results for anti-reflux surgery are generally good, however, patients who have failed anti-reflux procedures constitute a particularly challenging group. Which of the following statement(s) is/are true concerning failed anti-reflux repairs?

a. A Slipped Nissen is usually the result of an operative technical mistake
b. Disruption of a fundoplication is more prone to occur with a Nissen fundoplication because of the use of the gastric wall in the repair
c. Postoperative dysphagia in a patient with normal preoperative motility is usually due to a secondary motility disorder
d. Colonic replacement, although technically challenging, usually has superior long-term results when compared to esophageal replacement with the stomach
Answer: a, d

When patients are correctly selected and the operation performed with conformity with the basic surgical principles, long-term relief of symptoms is achieved by more than 90% of patients. A number of patterns of failure, however, can occur. The so-called Slipped Nissen may develop when the upper stomach rides up through the fundoplication, and causes both dysphagia and heartburn. It is more likely that the condition was created at the time of surgery because the surgeon did not mobilize the fundus, or because unrecognized esophageal shortening led to inadequate mobilization of the gastroesophageal junction, causing the surgeon to wrap the stomach around the upper stomach rather than the lower esophagus. Creating too tight a fundoplication leads to immediate postoperative dysphagia. Manometry shows a high pressure nonrelaxing sphincter which may be difficult to distinguish from achalasia. Such patients highlight the importance of manometry in all patients before proceeding with anti-reflux surgery. In a patient with normal preoperative motility, the cause is usually a fault in technique, and can be prevented by constructing the fundoplication over a 60 F Bougie. Disruption of the fundoplication that manifests clinically and physiologically by recurrent reflux can be caused by inadequate suture technique, unrecognized esophageal shortening leading to tension on the wrap, or poor choice of operation. All partial fundoplications, such as the Toupet procedure are more prone to disruption than a Nissen. This is because the integrity of the repair depends on sutures to the esophageal wall and not the stomach, and because all these repairs require much more abdominal length of esophagus than a Nissen, thus placing the repair under tension. Esophagectomy and esophageal replacement are occasionally indicated in the treatment of advanced GERD. The indications for esophagectomy are Barrett’s esophagus with high grade dysplasia and what is generally described “burned out esophagus” which includes failure of a third anti-reflux operation, a severe coexistent motility disorder, or the presence of an undilatable stricture. Either colon or stomach may be used to replace the esophagus. Colonic replacement is more difficult, requiring three anastomoses rather than one, but it has superior functional long-term results.


60. A number of diagnostic modalities exist for investigation of structural abnormalities of the esophagus. Which of the following statement(s) is/are true concerning the use of these investigative studies?

a. Endoscopy should be the first investigation in any patient with foregut symptoms
b. Barrett’s esophagus is suggested when the squamo-columnar junction is more than 2 cm above the gastroesophageal junction on endoscopic examination
c. There are three areas of esophageal narrowing which can be noted on both barium esophogram and endoscopy
d. The CT appearance of the esophagus is normally a flattened, hollow structure with a thin wall
Answer: b, c, d

Endoscopy is generally the first investigation in patients with foregut symptoms. The exception is when the patient’s chief complaint is dysphagia, when a “road map” should first be obtained by a barium swallow. The locations of the esophageal landmarks are measured endoscopically from the incisor teeth. Three landmarks are measured in the region of the cardia: the level of the crura, the level of the anatomic gastroesophageal junction, and the level of the squamo-columnar junction (Z line). A hiatal hernia is present when the gastroesophageal junction is more than 2 cm above the crura. Barrett’s esophagus is suggested when the squamo-columnar junction is more than 2 cm above the gastroesophageal junction but may be diagnosed if any specialized epithelium is identified above the gastroesophageal junction histologically, regardless of measured length of the columnar segment. Three areas of esophageal narrowing are frequently noted on both barium esophogram and endoscopy. The first narrowing is at the site of the cricopharyngeus muscle. The left mainstem bronchus and aortic arch caused narrowing of the middle third of the esophagus. The most distal narrowing of the esophagus is at the diaphragmatic hiatus and is caused by the lower esophageal sphincter mechanism. These normal points of narrowing tend to retard swallowed foreign objects. Also, corrosive liquid ingestion results in prominent mucosal injury at these sites as the liquid is slowed at passage. CT scan of the esophagus is important in delineating the relationship of esophageal lesions to adjacent structures, especially the trachea, left main bronchus and aorta. The esophagus normally appears as a flattened hollow structure with a thin wall. A more circular cross-sectional appearance with a fluid level is evidence of distal obstruction.


61. Which of the following patient scenarios would be best managed with anti-reflux surgery?

a. A patient with heartburn but normal 24 hour pH monitoring and an intact lower esophageal sphincter
b. A patient with primarily respiratory manifestations of gastroesophageal reflux
c. A patient with increased acid exposure and a mechanically defective sphincter who responds well to medical therapy but requires continued long-term medication for continued relief
d. A patient with gastroesophageal reflux but excessive complaints of epigastric pain, nausea, vomiting, and loss of appetite
Answer: b, c

The first requirement for consideration of anti-reflux surgery is the objective demonstration of the presence of GERD by 24-hour pH monitoring. Secondly, the patient must have either symptoms or complications of the disease. Thirdly, the disease should be caused by defect appropriate to surgical therapy, i.e., a mechanically defective sphincter. Some patients with increased acid exposure and a mechanically defective sphincter, and who have no complications of the disease respond well to medical therapy, but they require long-term medication for continued relief. These patients should be given the option of surgery as a cost effective alternative.
Atypical symptoms of reflux such as respiratory manifestations often respond well to anti-reflux surgery. When respiratory symptoms are combined with typical symptoms such as heartburn and regurgitation, the results of anti-reflux surgery are generally good.
Complaints of epigastric pain, nausea, vomiting, and loss of appetite may be due to excessive duodenogastric reflux which occurs in about 11% of patients with gastroesophageal reflux disease. This problem is usually, but not invariably, confined to patients who have previous upper gastrointestinal surgery. The coexistence of these gastric symptoms in a patient who also has typical symptoms of GERD should prompt a thorough evaluation of the stomach using a bile probe, 24 hour pH monitoring or radionucleotide scanning. In such patients, the correction of only the incompetent cardia can result in a disgruntled individual who continues to complain of nausea and epigastric pain on eating.


62. Which of the following statement(s) concerning pharyngoesophageal disorders is/are true?

a. In neuromuscular diseases, dysphagia is often worse for liquids than for solids
b. Cricomyotomy may be indicated for a wide variety of neuromuscular disorders involving the pharyngoesophageal phase of swallowing
c. Excision of a Zenker’s diverticulum is indicated to prevent malignant change in the sac
d. Complications of all operations on the cervical esophagus include hematoma formation and recurrent nerve paralysis
Answer: a, b, d

Disorders of the pharyngoesophageal phase of swallowing result from a discoordination of the neuromuscular events involved in chewing, initiation of swallowing, and propulsion of the material from the oropharynx to the cervical esophagus. The commonest causes of pharyngoesophageal dysphagia are neuromuscular diseases. The most important are cerebrovascular disease, myasthenia gravis, Parkinson’s disease, multiple sclerosis and muscular diseases such as myotonic dystrophy and polymyositis. In neuromuscular diseases, dysphagia is often worse for liquids than for solids. Choking, repetitive pneumonia, nasal regurgitation and hoarseness are also prominent features. The surgeon’s role in the treatment of cricopharyngeal disorders is to reduce outflow resistance by performing a cricomyotomy. Initially this was recommended only for patients with demonstrable failure of the upper esophageal sphincter relaxation. More recently, a number of reports indicate a wide variety of neuromuscular diseases that may be improved by cricomyotomy. The surgical options in Zenker’s diverticulum are either excision or suspension. Excision is sometimes recommended on the grounds that malignant change in the sac is prevented, but there is no evidence that excision carries any greater protective role than suspension, which effectively prevents stagnation of food material, thus removing the presumed cause of malignant change. Suspension also removes the risk of contamination of the operative site, the risk of subsequent breakdown of the closure site with fistula formation, and the risk of narrowing of the esophagus. In either case, recurrence is likely if cricomyotomy is not performed, because the underlying defect which predisposes to the diverticulum persists. All operations on the cervical esophagus carry the risk of hematoma formation and recurrent nerve paralysis. The venous pumping action of the lung can cause the development of a large hematoma in the mediastinum postoperatively, therefore meticulous hemostasis is critical for the performance of this operation.


63. Barrett’s esophagus is a complication of gastroesophageal reflux disease. Which of the following statement(s) is/are true concerning this condition?

a. The histologic hallmark is the presence of “specialized” columnar epithelium regardless of how far it extends into the esophagus
b. Barrett’s epithelium will frequently regress with medical therapy or anti-reflux surgery
c. High grade dysplasia will frequently be associated with foci of invasive carcinoma
d. Patients with adenocarcinoma arising in Barrett’s esophagus have a high incidence of p53 gene mutations
Answer: a, c, d

Barrett’s esophagus is now recognized as a complication of advanced gastroesophageal reflux disease. The histologic hallmark of Barrett’s esophagus is the presence of “specialized” columnar epithelium, which shows features of intestinal metaplasia, easily recognized by the presence of goblet cells. The presence of specialized epithelium is now regarded as the pathonomonic feature of Barrett’s esophagus regardless of how high it extends into the esophagus. Barrett’s esophagus may exist on its own, or may be itself associated with ulceration, stricture, and malignant change. Once Barrett’s epithelium is present, medical therapy or anti-reflux surgery rarely causes it to regress. Unless it is actually ablated, for example with laser therapy, it persists. The most significant feature of Barrett’s esophagus is its malignant potential. The metaplastic epithelium usually undergoes dysplastic change prior to becoming frankly neoplastic. High grade dysplagia is synonymous with carcinoma in situ, and if the esophagus is removed for such a condition, up to 50% will demonstrate foci of invasive carcinoma.
In the past, the pathophysiology of Barrett’s esophagus was associated with alkaline reflux on esophageal pH monitoring. However, more recently using a bile sensor for monitoring bilirubin, this condition is frequently associated with excessive bile in the esophagus. Repetitive injury from noxious gastric juice can lead during the repair process to mutations in the p53 gene. Patients with adenocarcinoma arising in Barrett’s esophagus have a high incidence of p53 mutations.


64. Which of the following statement(s) is/are true concerning the blood supply and lymphatic drainage of the esophagus?

a. The thoracic esophagus receives no direct branches from the aorta therefore allowing the technique of transhiatal (blunt) esophagectomy
b. Bleeding esophageal varices are most prominent in the mid-esophagus
c. Lymphatic drainage of the lower third of the esophagus goes entirely to the abdominal lymphatic system
d. Nodal involvement in esophageal cancer is quite common even if the tumor is limited to the level of the submucosa
Answer: d

The blood supply and venous drainage of the esophagus are largely segmental. The inferior thyroid artery provides the main blood supply to the cervical portion of the esophagus. The thoracic portion of the esophagus receives its blood supply from two sources; branches from two or three bronchial arteries provide the proximal arterial supply and branches directly from the aorta supply the more distal thoracic esophagus. Intrathoracic mobilization of the esophagus during performance of anti-reflux procedures often require ligation of these branches. The venous plexus in the submucosa collects capillary blood and delivers it into a periesophageal venous plexus. The left gastric vein or coronary vein provides the principal collateral in portal hypertension when esophageal varices develop. The submucosal veins become much more superficial in the most distal esophagus, 1–2 cm above the gastroesophageal junction, and are consequently the most common site of bleeding in portal hypertension.
The lymphatics of the esophagus form a rich submucosal network draining into regional lymph nodes in the periesophageal connective tissue. There is thus little barrier to longitudinal spread of cancer in the esophagus. Lymphatic drainage from the upper two-thirds of the esophagus is usually cephalad, but drainage from the lower one-third is in both directions. Although lymphatic metastasis in the esophagus generally involve the regional lymph nodes in proximity, nodal involvement may occur several centimeters away from the primary lesion because of the rich intramural lymphatic anastomotic channels. When a carcinoma is limited to the mucosa, the incidence of lymphatic metastases is low, but once into the submucosa, the incidence rises to 60%.


65. Which of the following statement(s) is/are true concerning the process of swallowing and esophageal transit of food?

a. Injury to the recurrent laryngeal nerves can cause motility problems of the cervical esophagus and resulting aspiration
b. Esophageal reflux does not lead to impaired esophageal motility
c. Relaxation of the LES is mediated via inhibitory neurons
d. The overall length of the LES is the only factor influencing the pressure gradient of the sphincter
e. A mechanically defective sphincter is always associated with increased esophageal acid exposure
Answer: a, c

The cricopharyngeus muscle is a continuation of the inferior constrictor of the pharynx and receives its innervation via both the right and left recurrent laryngeal nerves. Although much attention is given to vocal cord dysfunction that accompanies recurrent laryngeal nerve damage, it is clear that cricopharyngeal sphincter dysfunction and motility problems of the cervical esophagus can occur with injury to these nerves. Serious aspiration following recurrent nerve injury is caused not only by the cricopharyngeal dysfunction, but also by additional morbidity incurred because of the inability to close the glottis during swallowing and loss of the protection afforded by effective coughing. Clinically, peristaltic defects of the esophageal body fall into one of to broad categories. One category is characterized by a defect in organization of peristaltic waves, and is primarily a neural phenomenon. The other notable defect is reduction of the power (amplitude) of peristalsis and is usually due to muscle damage secondary to severe reflux or replacement with fibrous tissue as happens in scleroderma and other connective tissue diseases or with severe reflux. The LES provides a pressure barrier between the esophagus and stomach. The sphincter normally remains actively closed to prevent reflux of gastric contents into the esophagus. Relaxation of the LES is mediated by inhibitory neurons. It occurs either to allow entry of food, or to allow exit of air during belching. The ability of the LES to remain closed in the face of a pressure gradient tending to promote reflux of gastric contents from the positive pressure environment of the stomach into the negative pressure environment of the chest depends on several features. The most significant is the resting pressure. However, of equal importance is the ability of the LES to respond to variations in intra-abdominal pressure associated with daily activities. Such elevations would normally be transmitted to the sphincter, causing it to collapse and remain closed, provided sufficient length of the sphincter remains exposed to the abdominal pressure and the compressive effect of the crura. The abdominal length is often reduced in hiatal herniation, because of attenuation of the pharyngoesophageal membrane. The overall length of the LES is also an important determinant of competence, much as the total resistance of a series of resisters in a circuit is the sum of the individual resistances. A mechanically-defective sphincter, however, is not always associated with increased esophageal acid exposure because it may be compensated by the clearance function of the esophageal body. The role of the esophageal body in limiting acid reflux is related to its ability to clear the esophagus of acid. This clearance has two components: volume clearance which requires peristalsis, and chemical clearance which requires saliva.


66. Which of the following statement(s) is/are true concerning the management of this patient?

a. The risk of perforation of the esophagus associated with balloon dilatation may be as high as 10%
b. An anti-reflux procedure should be universally performed for any operative myotomy
c. Successful relief of dysphagia can be achieved in up to 90% of patients with a single pneumatic dilatation
d. Thoracoscopic myotomy is associated with significantly poorer results than the open procedure
e. Prospective randomized studies and retrospective data appear to support a surgical approach for achalasia
Answer: a, e

The mainstay of treatment in achalasia is either balloon dilatation or surgery. Balloon dilatation has an advantage that it can be performed as an outpatient and has minimal recovery time. It is less likely to be effective than surgical treatment, and frequently needs to be repeated. The risk of perforation of the lower esophagus is higher with this procedure than with any other form of esophageal instrumentation and varies from 2–10%. The risk of gastroesophageal reflux following dilatation is not known, but symptomatically the risk appears to be low.
All surgical procedures employ a variant of Heller’s myotomy, in which the circular muscle of the lower esophagus is divided. In the United States, most myotomies are carried out through the chest, but the abdominal approach is favored in Europe. Regardless of the route chosen, four key principles are important, namely: 1) adequate myotomy, 2) minimal hiatal disturbance, 3) anti-reflux protection without creation of obstruction, and 4) prevention of rehealing. The advent of minimally invasive surgery has led to the development of thorascopic and laparoscopic myotomy, and these are now being extensively performed with comparable results to open surgery. There is broad agreement that if a myotomy is performed through the abdomen, an anti-reflux procedure should be added, and that a full Nissen wrap, however floppy, leads to long-term failure. When approached through the chest, there is controversy about the need for an anti-reflux procedure, as it is claimed that less hiatal disturbance and more limited myotomy is possible by this route. Thoracoscopic myotomy, with enhanced view, enables a more precise determination of the distal myotomy and therefore may not require a anti-reflux procedure.
A single pneumatic dilatation achieves adequate relief of dysphasia and pharyngeal regurgitation in about 60% of patients. Repetitive dilatations increase this figure to about 70%. Only one controlled randomized study comparing the two modes of therapy has ever been performed. The results of this study as well as a number of retrospective studies would appear to support operative myotomy as the initial treatment of choice.


67. Which of the following statement(s) is/are true concerning the surgical anatomy of the esophagus?

a. Surgical exposure of the cervical esophagus is best gained via the right neck
b. Spontaneous esophageal perforation tends to be associated with leakage into the left chest
c. Access to the entire thoracic esophagus can be obtained only via the left chest
d. The lower esophageal sphincter can be recognized distinctly by inspection of the gastroesophageal junction
Answer: b

A detailed knowledge of the relations of the esophagus is essential for the surgeon to be able to identify the site and significance of lesions seen by indirect studies such as endoscopy, contrast radiography, and CT scanning, as well as the safe performance of surgical procedures. The cervical esophagus is about 5 cm long. It begins at the level of C6 and extends to the lower border of T1, curving slightly to the left in its descent. Consequently, although the surgical approach to this portion of the esophagus may be from either side of the neck through an incision along the anterior border of the sternocleidomastoid muscle, the left side is chosen if possible. Above the level of the tracheal bifurcation, the esophagus moves to the right of the descending aorta. It then moves to the left, passes behind the tracheal bifurcation and the left main bronchus and descends to the diaphragm. In the lower third, the esophagus courses anteriorly and to the left to pass through the diaphragmatic hiatus. The lower esophagus is covered only by a flimsy mediastinal pleura on the left, and it is this portion which is most commonly the site of spontaneous perforation in Boerhaave’s syndrome. In general, the lower esophagus is most easily approached through the left chest, but access to the supra-aortic esophagus is restricted. Thus, a left thoracotomy is most useful for performing procedures involving the lower esophagus. However, access to the entire thoracic esophagus can be obtained only from the right chest. This incision, however, limits access to intraabdominal organs by the position of the liver and therefore normally requires a separate upper abdominal incision. The abdominal esophagus begins as the esophagus enters the abdomen through the diaphragmatic hiatus. It is surrounded by a fibroelastic membrane, the phrenoesophageal ligament which arises from the subdiaphragmatic fascia. The lower limit of the pharyngoesophageal membrane anteriorly is marked by a prominent fat pad, which corresponds to the gastroesophageal junction. The lower esophageal sphincter (LES) is a zone of high pressure 3–5 cm long at the lower end of the esophagus. Although it does not correspond to any macroscopic anatomical structure, its function appears to be related to the microscopic architecture of the muscle fibers.


68. Which of the following statement(s) is/are correct concerning the patient whose barium esophogram is shown below?

a. The patient’s complaint would be primarily chest pain and to a lesser degree dysphagia
b. The pathognomic feature of manometry is the presence of prolonged high amplitude waves
c. The patient will likely experience nutritional problems
d. The first line of treatment for this patient is surgical myotomy
Answer: a, b

The barium esophogram of these two patients shows diffuse esophageal spasm resulting in a cork screw esophagus with multiple contractions. See Fig. 18-35. These primary motor disorders are characterized by substernal chest pain. In the nutcracker variety, as demonstrated in this x-ray, the pain is central crushing pain with no relation to food ingestion and differs from angina in that it more frequently comes on at rest. Dysphagia or classic heartburn may be present but tend to be overshadowed by the chest pain. Barium radiography and endoscopy are generally not helpful. The pathognomic feature of manometry is the presence of prolonged high amplitude waves, with a peak greater than 180 mm Hg. Diffuse esophageal spasm and nutcracker esophagus are benign conditions which rarely cause nutritional problems and do not lead to life-threatening complications. For this reason, symptom control is the only significant goal of treatment. Medical treatment for diffuse esophageal spasm and nutcracker esophagus is focused on abolishing strong simultaneous contractions and generally employs calcium channel blocking agents or long-acting nitrates. Surgery for these conditions are generally only considered when medical treatment is ineffective.


69. Which of the following statement(s) is/are true concerning tracheoesophageal fistulas?

a. The majority of acquired tracheoesophageal fistulas are due to malignant disease
b. A water-soluble contrast esophogram should be obtained for diagnosis
c. Malignant tracheoesophageal fistulas represent one of the few indications for an endoesophageal prosthesis
d. A benign tracheoesophageal fistula from an endotracheal intubation injury often requires a thoracotomy for repair
Answer: a, c

Ninety percent of acquired fistulas between the esophagus and tracheobronchial tree in adults are the result of malignant disease. Tracheoesophageal fistulas complicate the course of disease in about 5% of patients who have esophageal carcinoma. Nearly 80% of patients with malignant tracheoesophageal fistulas die within three months of the onset of symptoms and in 85% of these patients, the cause of death is aspiration pneumonia, not distant metastatic disease. For the most part, malignant tracheoesophageal fistula represents incurable disease for which resection carries significant mortality and is seldom indicated. Palliative relief of recurrent aspiration is the aim of therapy. Effective occlusion of the fistula may be achieved by insertion of one of a variety of available endoesophageal endoprostheses. These tubes are placed into the esophagus with the aid of an esophagoscope and may occlude the esophageal side of the fistula sufficiently to allow swallowing of liquids without aspiration into the tracheobronchial tree. More recently, expandable metal stents have been used successfully in the treatment of malignant tracheoesophageal fistulas.
Nonmalignant fistulas result from the erosion by contiguous infected subcarinal mediastinal lymph nodes; trauma; late sequelae of chronic mid-esophageal traction diverticulum; or erosion by an endotracheal or tracheostomy tube cuff in a patient requiring prolonged ventilatory support. Small fistulas, such as resulting from an endotracheal intubation injury, are approached through a cervical collar or oblique incision anterior to the sternocleidomastoid muscle. Although such cuff injuries usually produce circumferential tracheal damage which necessitates a tracheal resection, this can also be performed through a cervical collar incision.


70. Esophageal cysts arise as outpouchings of the embryonic foregut. Which of the following statement(s) is/are true concerning esophageal cysts?

a. The cyst lining will be lined only by stratified squamous epithelium
b. Most esophageal cysts cause symptoms in the first year of life
c. An asymptomatic esophageal cyst can be managed conservatively
d. The diagnosis of an esophageal cyst is usually made radiographically
Answer: b, d

Embryologically, the esophagus is lined by simple columnar ciliated epithelium, which is eventually replaced by stratified squamous epithelium. The esophageal cyst may therefore contain both of these types of epithelium as well as fat and smooth muscle. The esophageal duplication cyst is a variation of the foregut cyst, extends along the length of the thoracic esophagus, and is lined by squamous epithelium. More than 60% of esophageal cysts cause either respiratory or esophageal symptoms in the first year of life. Those located in the upper third of the esophagus tend to present in infancy, while the lower-third cyst may be asymptomatic initially and present later in childhood. Adults present with dysphasia, choking, retrosternal pain when previously asymptomatic cysts enlarge as the result of bleeding or infection. The diagnosis of an esophageal cyst can usually be made on the basis of atypical radiographic appearance. The PA and lateral chest x-ray, barium esophogram, and in some cases a CT scan, will confirm the diagnosis in almost all patients. Because esophageal cysts have a predilection for bleeding, ulceration, perforation, and infection, excision is generally recommended. This can generally be achieved with low morbidity by an extramucosal resection.


71. Which of the following statement(s) is/are true concerning infectious esophagitis?

a. Candida albicans is not normally found in the mouth but results from the overgrowth of this fungus in patients on broad spectrum antibiotics
b. Candida esophagitis is usually self-limited and is seldom associated with chronic problems
c. Systemic therapy is seldom indicated
d. Small ulcers on barium esophogram in a transplant patient complaining of dysphagia and odynophagia are likely due to herpes simplex viral infection
Answer: d

Chronic debilitation, immunosuppression, and prolonged use of antibiotics predisposes the development of infectious esophagitis with candida albicans being the most common cause. Candida albicans is a fungus that normally is a commensal inhabitant of the mouth, oral pharynx, and GI tract. This fungus may become pathogenic in patients who are severely debilitated or immunosuppressed. In recent years, the use of broad spectrum antibiotics, immunosuppression in organ transplant patients, and the wide use of chemotherapeutic agents have resulted in an increased number of patients with monilial esophagitis. As the disease progresses, transmural invasion of the esophageal wall occurs. Although the esophagitis can be controlled with antifungal therapy, if the patient survives the underlying illness, chronic stricture formation may result after healing. Minimally compromised patients with mild monilial esophagitis should receive oral nystatin suspension as a primary treatment. More immunosuppressed patients or those with severe cases warrant high doses of fluconazole and ketoconazole. Intravenous fluconazole or amphotericin B are utilized in granulocytopenic patients.
Viral esophagitis is the second most common cause of infectious esophagitis. Herpes simplex viral infection is the most common infection in the immunosuppressed transplant patient. Characteristically, viral esophagitis produces mucosal ulceration with patients presenting with dysphasia and odynophagia. The esophageal ulcers are characteristically small (< 1.5 cm). The diagnosis is established endoscopically by biopsy, brushings, and washings for cytology, histology, and viral culture. The infection usually responds well to treatment with acyclovir.


72. Which of the following statement(s) is/are true concerning the pathology of squamous cell carcinoma of the esophagus?

a. Carcinoma in situ will gradually progress to invasive squamous cell carcinoma over a period of two to four years
b. The most common location for squamous cell carcinoma of the esophagus is the upper and mid-thoracic segment
c. Esophageal carcinoma tends to be multifocal
d. Macroscopically, ulcerative lesions with extensive infiltration of the adjacent esophageal wall are most common
e. Lymph node metastases are present in at least 75% of patients at the time of initial diagnosis
Answer: a, b, d, e

Pathologically, esophageal carcinoma occurs over a spectrum that ranges from the early lesion (carcinoma in situ), which is limited to the mucosa, to the more advanced form, in which the tumor penetrates the muscle layers of the esophagus and beyond. Carcinoma in situ typically is found in patients between 40 and 50 years of age and gradually progresses to invasive squamous cell carcinoma over two to four years. Using the arbitrary division of the esophagus, 8% of squamous cell carcinomas occur in the cervical esophagus, 55% in the upper and mid-thoracic segments, and 37% in the lower thoracic segment which extends to the GE junction. Macroscopically, 60% of squamous cell carcinomas of the esophagus are fungating intraluminal growths, 25% of ulcerative lesions are associated with extensive infiltration of the adjacent esophageal wall, and 15% are infiltrating. Esophageal carcinoma tends to be multi-focal, and a patient who survives treatment of one carcinoma has at least twice the risk of developing a second primary esophageal neoplasm than the normal population.
Esophageal carcinoma is notorious for its aggressive biologic behavior. Mediastinal, supraclavicular, or celiac lymph node metastases are present in at least 75% of patients with esophageal cancer at the time of initial diagnosis. Unfortunately, when lymph node metastases are present, five-year survival is only 3%, compared with 42% when there is no lymph node spread.


73. A 54-year-old woman experiences pain in both the anterior and posterior left chest and the epigastrium following balloon dilatation performed for achalasia. Which of the following statement(s) is/are true concerning this patient’s diagnosis and management?

a. A normal chest x-ray will rule out an esophageal perforation
b. Barium should never be used in performance of a contrast study with a diagnosis of esophageal perforation
c. Conservative, nonoperative treatment may be indicated
d. If surgical repair is necessary, the patient should undergo esophagomyotomy and a partial gastric fundoplication
Answer: c, d

It is axiomatic that pain or fever after esophageal instrumentation or operation is indicative of an esophageal perforation until proven otherwise. It is an indication for immediate contrast esophogram. Because the morbidity and mortality rates associated with esophageal perforation are directly related to the time interval between diagnosis of the injury and its repair or drainage, an aggressive attitude toward diagnosing the perforation must be adopted. When the diagnosis is considered, a water-soluble contrast agent should be administered. If this study is negative, dilute barium should be administered. Barium is relatively inert, and the fear of barium extravasating in the mediastinum through the site of injury and producing a severe reactive mediastinitis is unfounded. A chest x-ray may help confirm the diagnosis by demonstrating air in the soft tissues of the neck or mediastinum or a hyrdo-or pneumothorax. A normal chest x-ray, however, does not rule out an esophageal perforation.
Although most esophageal perforations require operative intervention, selected patients may be managed nonoperatively with cessation of oral intake, administration of antibiotics, and intravenous hydration until the disruption heals or the small contained cavity begins to decrease in size. The usual clinical settings for such perforations that are encountered are cervical esophageal tears caused by esophagoscopy; intramural dissection that has occurred during dilatation of a stricture or pneumatic dilatation for achalasia, and an asymptomatic esophageal anastomotic disruption discovered on a routine postoperative contrast study. Perforations complicating pneumatic dilatation for achalasia occur in 4% to 6% of patients, and most are small and well managed medically with antibiotics and intravenous hyperalimentation. If operation is required for suture repair of the perforation, an esophagomyotomy to relieve the distal obstruction, and a partial fundoplication to buttress the tear should be performed if possible.


74. Which of the following statement(s) is/are correct concerning the diagnostic studies for esophageal carcinoma?

a. A chest and upper abdominal CT scan is useful for both staging and predicting resectability
b. A barium swallow is an unnecessary test in a patient with dysphagia
c. Bronchoscopy should be performed in all patients with carcinoma of the upper and middle thirds of the esophagus
d. Bone and brain scans should be obtained routinely to rule out distant metastasis
e. Endoscopic ultrasound is a potentially sensitive examination for the staging of esophageal cancer
Answer: c, e

A barium swallow examination is the first study that should be obtained in a patient who complains of dysphagia. Tumors of the cervical esophagus are difficult to identify by barium swallow examination and carcinoma of the cardia may be confused with achalasia or esophageal spasm. Nevertheless, the barium swallow examination localizes obvious esophageal pathology in preparation for subsequent esophagoscopy and allows the endoscopist to predict the level at which the tumor is located and the area which requires the most careful examination. The chest and upper abdominal CT scan is now the standard radiographic technique for staging esophageal carcinoma. Esophageal wall thickness, regional adenopathy or pulmonary, liver, adrenal or distant nodal metastasis can be identified. Although CT is suggested to have a role in evaluating resectability of esophageal carcinoma, it is particularly limited in its ability to detect invasion of the gastric cardia or aortic invasion. Bone scan is not warranted unless the patient has specific complaints suggesting that bone metastases exists. Similarly, routine brain scans are not indicated as brain metastases from carcinoma of the esophagus are uncommon. Bronchoscopy should be performed in patients with carcinoma of the upper and middle thirds of the esophagus to exclude invasion of the posterior membranous trachea or mainstem bronchi, which precludes a safe esophagectomy. Endoscopic ultrasound is being used with increasing frequency as an adjunct to the standard radiologic and endoscopic assessment of esophageal disease. It offers the potential for more sensitive staging of esophageal carcinoma by detecting the depth of invasion and the presence of abnormal mediastinal adenopathy.


75. Which of the following conditions are associated with the development of esophageal carcinoma?

a. Caustic esophageal stricture
b. Achalasia of the esophagus
c. Plummer-Vinson syndrome
d. Esophageal diverticula
Answer: a, b, c, d

Chronic irritation of the esophageal mucosa by a variety of noxious stimuli (alcohol, tobacco, hot foods and liquids) eventually may lead to the development of esophageal carcinoma. A variety of other esophageal lesions have a recognized premalignant nature. The patient who survives the initial injury long enough to develop a caustic esophageal stricture has a 1000-fold increased risk of developing carcinoma compared with the normal population. Ten to 12% of patients with achalasia of the esophagus who are observed 15 years or more develop esophageal carcinoma. This is thought to be related to the irritating effects of the fermenting intraesophageal contents on the adjacent esophageal mucosa. Plummer-Vinson syndrome is a premalignant esophageal condition. Patients with this syndrome are typically elderly women who have cervical dysphasia and iron deficiency anemia. About 10% of patients will develop squamous cell carcinoma of the hypopharynx, oral cavity or esophagus. Finally, there have been isolated reports of esophageal carcinomas found incidentally within esophageal diverticula, presumably as the result of the irritating effects on the mucosa of stagnant, putrefying food within the pouch. Esophageal diverticula are therefore regarded as premalignant esophageal lesions although this occurrence is extremely rare.


76. Which of the following statement(s) is/are correct concerning the options for resection of esophageal carcinoma?

a. The development of reflux esophagitis seldom occurs following intrathoracic resection due to the limited life expectancy of these patients
b. Transhiatal esophagectomy, although conceptually sound, is not technically possible in most patients with esophageal carcinoma
c. Transhiatal resection, although less morbid, has unfavorable survival statistics compared to transthoracic resection
d. Radical transthoracic esophagectomy with en bloc dissection of continuous lymph node bearing tissues has not been shown to improve survival over transhiatal esophagectomy
Answer: d

For most patients with localized esophageal carcinoma, resection provides the most effective and reliable palliation of dysphagia. The rational surgical approach to distal esophageal carcinoma has been a left thoracoabdominal incision. Tumors involving the mid-esophagus have been resected either through a thoracoabdominal or separate thoracic and abdominal incision, and a high thoracic esophagogastric anastomosis is performed. The major disadvantages of this technique are the necessity of a thoracotomy in debilitated patients with esophageal obstruction as well as the disastrous complications following intrathoracic esophageal anastomotic leak. Although recent results have shown improved operative mortality rates, the operation can still be associated with significant morbidity and mortality. A further disadvantage of the standard intrathoracic esophagogastric anastomosis is inadequate long-term relief of dysphagia either due to tumor recurrence at the anastomotic suture line or due to the development of reflux esophagitis above the anastomosis. Although it has been long taught that the patient with esophageal carcinoma does not live long enough to develop reflux esophagitis after a low intrathoracic esophagogastric anastomosis, this is clearly not the case, and the development of reflux in these patients can produce not only severe pyrosis and reflux symptoms, but also dysphagia from benign stenosis.
During the last two decades, the technique of transhiatal esophagectomy without thoracotomy has been popularized as an operation that minimizes the factors responsible for poor results from traditional transthoracic esophageal resection and reconstruction. In experienced hands, transhiatal esophagectomy without thoracotomy is possible in over 90% of patients. The survival data is comparable to those obtained in most series of transthoracic resection with results usually demonstrating decreased postoperative morbidity and mortality. Although conceptually radical transthoracic esophagectomy with en bloc dissection of contiguous lymph node bearing tissues would appear to offer a better “cancer operation” than transhiatal esophagectomy with no formal lymph node dissection, current survival results are not statistically different. These data suggest that survival after resection for esophageal carcinoma is more a function of the extent and stage of the tumor rather than the size of the specimen or the number of lymph nodes removed.


77. The incidence of adenocarcinoma of the esophagus is increasing at a very rapid rate, which is largely the result of the growing prevalence of adenocarcinoma arising in Barrett’s mucosa. Which of the following statement(s) is/are true concerning adenocarcinoma of the esophagus?

a. Barrett’s mucosa with specialized columnar epithelium characterized by veliform folds, lined by secreting columnar and goblet cells has the highest association with carcinoma of the esophagus
b. Less than 5% of patients with Barrett’s mucosa will harbor adenocarcinoma
c. Severe dysplasia of Barrett’s mucosa requires frequent reexamination and biopsy
d. Adenocarcinoma of the esophagus has a less aggressive behavior than squamous cell carcinoma
Answer: a

It is estimated that patients with Barrett’s esophagus are 40 times more likely to develop adenocarcinoma than the general population. The true incidence of Barrett’s esophagus in the general population is unknown, but it is estimated that adenocarcinoma arises in up to 8% to 15% of patients with columnar epithelium lined esophagus. Of the three characteristic histologic patterns for Barrett’s mucosa, the specialized or intestinal type of metaplasia which is characterized by veliform folds lined by a single layer of glycoprotein secreting columnar cell and mucous-secreting goblet cells has the highest association with carcinoma. Dysplasia occurs to varying degrees in Barrett’s mucosa and is clearly a premalignant lesion. Severe dysplasia is almost always associated with carcinoma in situ and mandates aggressive therapy. As is true of squamous cell carcinomas, esophageal adenocarcinoma has an aggressive biologic behavior that is characterized by frequent transmural invasion and lymphatic spread.


78. Which of the following statement(s) is/are true concerning esophageal diverticula?

a. A Zenker’s diverticulum characteristically occurs in older patients
b. Mediastinal granulomatous disease usually results in a mid-esophageal traction diverticulum which is usually asymptomatic
c. An epiphrenic diverticulum that presents to the right of the esophagus should be managed via left thoracotomy
d. Minimally symptomatic epiphrenic diverticula should not be operated upon
Answer: b, c, d

An esophageal diverticulum is an epithelial-lined mucosal pouch that protrudes from the esophageal lumen. Most esophageal diverticula are acquired, and occur predominantly in adults. The pharyngoesophageal (Zenker’s diverticulum) is the most common esophageal diverticulum and typically occurs in patients between 30 and 50 years of age. Mediastinal granulomatous disease (e.g., tuberculosis or histoplasmosis) is the most common cause of mid-esophageal traction diverticulum. This type of diverticulum is much smaller than the pulsion diverticulum and has a characteristic blunt tapered tip that points toward the adjacent subcarinal or peribronchial lymph nodes to which it adheres. It is typically diagnosed as an incidental finding on a barium esophogram and is almost always asymptomatic. No specific treatment is indicated.
An epiphrenic or supradiaphragmatic diverticulum occurs within the distal 10 cm of the thoracic esophagus as a pulsion diverticulum that arises because of abnormally elevated intraluminal esophageal pressure. Although many patients are asymptomatic at the time of diagnosis on barium esophogram, others have symptoms from the frequently associated esophageal conditions: hiatal hernia, diffuse esophageal spasm, achalasia, reflux esophagitis and carcinoma. Pouches smaller than 3 cm and causing little or no symptoms require no treatment. Severe dysphagia, chest pain, or an anatomically dependent or enlarging pouch are indications for repair. The surgical approach to an epiphrenic diverticula is through a left 6th or 7th interspace posterolateral thoracotomy. This is the case even for diverticula that present to the right of the esophagus.


79. Which of the following statement(s) is/are true concerning caustic injury to the esophagus?

a. Alkaline injury is more destructive than acid injury
b. Acid ingestion is not injurious to the stomach due to its normal acidic pH
c. Ingested caustic agents rapidly pass through the esophagus and stomach into the small intestine
d. Unless perforation occurs, clinical manifestations resolve quickly with initial clinical improvement noted
e. Children are less likely to form a late esophageal stricture than adults
Answer: a, d, e

Caustic injury occurs in two broad categories of patients, children younger than 5 years of age who accidentally swallow these agents, and adults who are attempting suicide. The most common agents responsible for caustic esophageal injuries are alkalis, acids, bleach and detergents. Ingestion of detergents and bleaches virtually always cause only mild esophageal irritation which heals without significant adverse sequelae. Acids and alkalis may have devastating effects which range from acute multi-organ necrosis and perforation to chronic esophageal and gastric strictures. Alkalis are more destructive, producing liquefaction and necrosis which almost insures deep penetration whereas acids usually cause coagulation necrosis that, in part, limits the depth of injury. Liquid alkali preparations have prolonged contact with the mucosa of the esophagus and stomach due to its high viscosity. In addition, ingested acids typically pass through the esophagus quickly, producing major gastric injury with relative sparing of the esophagus. In response to either ingested acid or alkali, reflex pyloric spasm occurs, with resultant pooling of these agents in the gastric antrum.
The clinical manifestations of caustic ingestion are directly related to the amount and character of the agent ingested. When esophageal or gastric perforation results from caustic ingestion, patients demonstrate progressive severe sepsis and hypovolemic shock until appropriate resuscitative measures are instituted. In the absence of gastric or esophageal perforation, the acute clinical manifestations typically resolve within days, with clinical improvement lasting for weeks. After this, symptoms due to either esophageal or gastric stricture begin to form. Most adults who ingest liquid alkali will develop severe esophageal and usually gastric injury that results in stricture formation. Children, with usually more limited exposure from accidental ingestions, are less likely to have severe injuries.


80. Which of the following statement(s) is/are true concerning nonresectional therapy for esophageal carcinoma?

a. Radiation therapy can be associated with five-year survival rates equal to surgery
b. Esophageal intubation to provide palliation for esophageal cancer is associated with minimal morbidity and mortality
c. Endoscopic laser fulguration is successful in up to 75% of patients
d. There is little or no role for surgical bypass for unresectable esophageal carcinoma
Answer: c, d

Therapy of esophageal carcinoma is influenced by the knowledge that in most of these patients, local tumor invasion or distant metastatic disease preclude cure. While squamous cell carcinoma is generally regarded as a radiosensitive and therefore potentially curable tumor, radiation therapy has not achieved cure in most patients. Although “curative” super voltage radiation techniques have been employed, the average five-year survival after such treatment is between 6 and 10% in most series. This is somewhat poorer than five-year survival rates after resection which usually range between 10 and 15%. A variety of endoesophageal tubes have been used for palliation in patients with esophageal carcinoma. Basically, these tubes are divided into two types: the pulsion tubes, which are pushed through the tumor with the aid of an esophagoscope; and the traction or pull-through tubes, which are pulled into place by downward traction through a gastrostomy. As in the case with many conceptually simple procedures, implementation in the clinical setting is problematic. Transoral esophageal intubation is associated with an overall mortality of 14% and a complication rate of 25%, the latter due to perforation of the esophagus, migration of the tubes, or obstruction of the tubes by food or tumor overgrowth. More recently, a variety of expandable intraesophageal metallic stents have been used to achieve palliation in patients with unresectable esophageal carcinoma. Additional expertise with this technique is being acquired through a multi-institutional trial currently underway in the United States. Endoscopic laser fulguration of esophageal carcinoma has been used to achieve temporary relief of the esophageal obstruction in patients with unresectable tumors. Generally, multiple sessions are required to resect sufficient tumor to achieve adequate lumen, but functional success with restoration of a comfortable volume can be achieved in 75–80%. Although a variety of surgical procedures such as substernal gastric or colon bypasses have been developed as palliative internal bypasses of unresectable esophageal carcinoma, the limited survival in these unresectable patients can be high and the mortality rates, between 15 and 25%, do not currently justify their use.


81. Benign tumors of the esophagus are rare constituting less than 1% of esophageal neoplasms. Which of the following statement(s) is/are true concerning benign esophageal neoplasms?

a. Most esophageal polyps are located just above the gastroesophageal junction
b. Malignant degeneration of leiomyomas of the esophagus is a frequent occurrence
c. An asymptomatic leiomyoma can be safely observed and followed with periodic barium esophograms and endoscopic ultrasonography
d. Most leiomyomas of the esophagus require esophagectomy
Answer: c

Leiomyomas represent the most common benign intramural esophageal tumor and characteristically occur in patients between 20 and 50 years of age. More than 80% of esophageal leiomyomas occur in the middle and lower thirds of the esophagus. Tumors less than 5 cm in size rarely cause symptoms. When larger than this, dysphagia, retrosternal pressure and pain are the common complaints. Bleeding more often occurs with the malignant form of the tumor, leiomyosarcoma. Malignant degeneration of leiomyomas is exceedingly rare. An asymptomatic leiomyoma or one discovered incidentally on a barium swallow examination can be safely observed and followed with periodic barium esophograms and endoscopic ultrasonography. Although excision of the esophageal mass provides the only definitive tissue diagnosis, the characteristic radiographic appearance, slow growth rate, and low risk of malignant degeneration and the ability to follow leiomyomas with endoscopic ultrasonography justify conservative management. Tumors that are symptomatic or larger than 5 cm in diameter should be excised. Tumors of the middle third of the esophagus are approached through a right thoracotomy, while those in the distal third are approached through a left thoracotomy. Once the esophagus is encircled and the tumor located, the overlying longitudinal muscle is split in the direction of its fibers. The tumor is then gently dissected away from the contiguous underlying submucosa and adjacent muscle. When enucleation of the tumor is complete, the longitudinal esophageal muscle is reapproximated, although a large extramucosal defect may be left without complication. Giant leiomyomas of the cardia and adjacent stomach may require esophageal resection for their removal. When resection is complete, leiomyomas virtually never recur.
Benign polyps of the esophagus are rare and typically arise in the cervical esophagus. Most are seen in older men and are frequently attached to the cricoid cartilage. Histologically, they are composed of fibrovascular tissue with varying amounts of associated fat.


82. In an effort to improve survival following esophageal resection, trials of multi-modality therapy in combination with surgery have been completed. Which of the following statement(s) is/are true concerning such treatment?

a. Therapy appears to be indicated in squamous cell carcinoma but not adenocarcinoma
b. No residual carcinoma may be found in the resected specimen in up to 20% of patients
c. Nonrandomized trials would suggest improved survival compared to patients receiving surgery alone
d. Perioperative morbidity is increased due to preoperative radiation and chemotherapy
Answer: b, c,

Combined preoperative chemotherapy and radiation therapy before transhiatal esophagectomy for carcinoma has provided encouraging survival statistics. The treatment consists of preoperative chemotherapy with three weeks of cisplatin, vinblastine, and 5-fluorouracil, concurrent with 3750 to 4500 cGy of radiation therapy. Although hematologic toxicity and radiation esophagitis are common and preoperative deaths due to bone marrow suppression can occur, there is no increase in perioperative morbidity when compared to patients with no preoperative therapy. In one study, 24% of patients had no residual carcinoma in the resected specimen (T0, N0 status). At mean follow-up of 36 months, the mean survival in this series was 29 months, a clear improvement over the 12-month median survival time with transhiatal esophagectomy alone in historical controls.


83. Which of the following statement(s) is/are correct concerning the management of a patient with a caustic esophageal or gastric injury?

a. Corticosteroids should be administered immediately
b. Complete endoscopic examination of the esophagus and stomach should be completed
c. Patients requiring operative intervention are best explored through the abdomen
d. If organ resection is indicated, restoration of alimentary continuity should be deferred until the patient has recovered from the acute insult
e. In patients with esophageal stricture following second and third degree burns, dilatation therapy should be instituted as soon as possible after the injury
Answer: b, c, d

Acute caustic ingestion is indication for hospitalization. Initial management centers on stabilizing the patient and assessing the severity of injury. Oral intake should be withheld and hypovolemia corrected with intravenous fluids. Careful observation for evidence of airway obstruction is mandatory. Broad spectrum antibiotics are indicated once the diagnosis of substantial esophageal injury has been established to diminish the risk of pulmonary infection from aspiration as well as bacterial invasion through the damaged esophageal wall. Although corticosteroids have been advocated in the acute phase of caustic ingestion to minimize subsequent stricture formation, their efficacy has not been established. Furthermore, because steroids may mask signs of sepsis, visceral perforation, and impair healing, their use in caustic esophageal injury is potentially deleterious and therefore is not recommended.
A contrast esophogram is the best way to make the diagnosis of esophageal perforation and should be performed if the diagnosis is suspected at the time of admission or in subsequent followup. Esophagoscopy should be performed soon after admission to establish whether significant esophageal injury has occurred and to permit grading of the severity of injury. Although in the past it was taught that the endoscope should not be advanced beyond the first burned area, more recently complete examination of the esophagus and stomach has been recommended, especially if severe burns are not detected proximally. The use of a pediatric endoscope and adequate sedation can allow this procedure to be accomplished safely. Patients with caustic liquid ingestion that necessitate operative intervention are generally best explored through the abdomen. This approach permits assessment of the injury to the intraabdominal organs as well as resection of areas of full thickness gastric necrosis. Although only the lower esophagus is well visualized through the diaphragmatic hiatus, if an esophageal resection is required, transhiatal esophagectomy without thoracotomy is readily performed by the addition of a cervical incision. When esophageal gastric resection for acute caustic injury is required, restoration of alimentary continuity should be deferred until the patient has recovered from the acute insult and the development of chronic stricture formation and retained organs can be evaluated. Esophageal stricture formation following second and third degree burns is the rule, and dilatation therapy has been the traditional therapy for chronic caustic esophageal strictures. It is important that dilatation not be instituted until at least six to eight weeks after the injury, when reepithelialization is complete, in order to minimize the risk of perforation.


84. In regard to the arterial blood supply to the stomach, which of the following statement(s) is/are true?

a. The right gastric artery, a branch of the superior mesenteric artery, supplies the gastric antrum
b. Because of rich intramural collaterals, gastric viability may be preserved after ligation of all but one major artery
c. In cases of celiac artery occlusion, gastric viability is maintained collaterally via pancreaticoduodenal arcades
d. The left gastroepiploic artery is a branch of the celiac trunk
Answer: b, c

The stomach is an extremely well-vascularized organ, supplied by 5 major arterial distributions and protected from ischemia by rich intramural and extramural collaterals. The left gastric artery and right gastric artery, derived from the celiac distribution, supply the lesser curvature of the stomach. The right gastroepiploic artery, derived from the gastroduodenal artery, and the left gastroepiploic artery, from the splenic artery, traverse the greater curvature. The area adjacent to the spleen receives multiple short gastric arterial branches. In instances of celiac arterial occlusion, the superior mesenteric artery supplies the stomach collaterally via the pancreaticoduodenal arcades which connect with the gastroduodenal artery. The stomach may be widely mobilized for use in reconstructive procedures, for example, during trans-hiatal esophagectomy. Advantage is taken of the abundant blood supply and collaterals of the stomach during mobilization; gastric viability is usually preserved if one major arterial supply is preserved.


85. At a cellular level, the major stimulant(s) of acid secretion by the gastric parietal cell is/are:

a. Histamine
b. Prostaglandin E2
c. Acetylcholine
d. Gastrin
e. Norepinephrine
Answer: a, c, d

The three major stimulants of acid secretion by the parietal cell are acetylcholine, gastrin, and histamine. Acetylcholine is released from cholinergic nerve endings in close proximity to parietal cells and binds to muscarinic receptors. Cholinergic stimulation of parietal cells is coupled to hydrolysis of membrane-associated lipids (termed phosphatidylinositides) and leads to increases in intracellular calcium. Histamine is released from mast cells in the lamina propria and reaches parietal cells by diffusion. Histamine occupies H2 receptors that may be selectively blocked by agents such as cimetidine. Histamine stimulation of parietal cell acid secretion is mediated by a cyclic AMP-dependent pathway. Gastrin is delivered to the fundic mucosa by the systemic circulation from its source in the antrum and duodenum. Like acetylcholine, gastrin causes increases in membrane phosphoinositol turnover and increases intracellular calcium.
Activation of parietal cells by acetylcholine, gastrin or histamine can be blocked by somatostatin. Local release of somatostatin is physiologically important in modulating postprandial gastric acid secretion. Prostaglandin E2 and its synthetic derivatives are potent inhibitors of histamine-stimulated acid secretion.


86. Which of the following statement(s) regarding the vagus nerves is/are true?

a. The right and left vagus nerves derive from a nerve plexus inferior to the tracheal bifurcation
b. The posterior vagus nerve is closely applied to the intrathoracic esophagus
c. The anterior vagus supplies a hepatic division which passes to the right in the lesser omentum
d. Approximately 90% of vagal fibers are afferent, transmitting information from the gastrointestinal tract to the central nervous system
e. The vagus nerves transmit gastroduodenal pain sensations associated with peptic ulceration
Answer: a, c, d

The left and right vagus nerves are formed from a periesophageal nerve plexus between the tracheal bifurcation and the diaphragm. As they pass through the esophageal hiatus, the anterior vagus is closely applied to the esophagus; the posterior vagus lies intermediate in position between the esophagus and the aorta. The anterior vagus supplies the hepatic division which provides parasympathetic innervation to the liver and biliary tract. The hepatic division is usually easily seen in the thin gastrohepatic omentum and is constant in location. The hepatic division is a useful anatomic landmark in vagotomy procedures. The posterior vagus nerve supplies fibers to the celiac division. After giving off hepatic and celiac divisions, both anterior and posterior vagus nerves supply branches to the gastric wall. Surprisingly, only 10% of vagal fibers are efferent, secretomotor fibers; almost 90% are afferent. Sensations of gastric pain are carried in sympathetic fibers, and vagotomy does not alter perception of painful gastric conditions or stimuli.


87. Important stimulants of gastrin release from endocrine cells in the antrum include:

a. Acidification of the antral lumen
b. Small peptide fragments and amino acids from luminal proteolysis
c. Locally released somatostatin
d. Dietary fats
Answer: b

Gastrin is processed to 34-and 17-amino acid forms in endocrine cells in the gastric antrum. In addition to well-recognized stimulatory actions on gastric acid secretion, gastrin promotes mucosal growth of the gastric fundus and small intestine. The most important stimulant of gastrin release is a meal. Small peptide fragments and amino acids that result from intragastric proteolysis are the food components that stimulate gastrin release. Ingested fats and carbohydrates have no significant effect. In this regard, intraluminal pH strongly affects gastrin secretion. If intragastric pH is maintained above 3 after ingestion of a meal, gastrin release is strongly potentiated. Pernicious anemia and atrophic gastritis, which produce chronic achlorhydria, are associated with fasting hypergastrinemia and an exaggerated gastrin meal response. Conversely, antral acidification strongly inhibits gastrin secretion. Locally-released somatostatin mediates the effects of luminal acidification, inhibiting gastrin secretion.


88. Which of the following statements regarding human gastric acid secretion is/are true?

a. Fasting acid secretion, normally 2 to 5 mEq/h, is due to ambient vagal tone and histamine secretion
b. Truncal vagotomy decreases basal secretion by 80%
c. Histamine2 receptor antagonist administration can decrease basal acid secretion by 80%
d. Fasting acid secretion, normally 5 to 10 mEq/h, is due to circulating levels of gastrin
Answer: a, b, c

Both vagal tone and locally secreted histamine are presumed to be the determinants of basal acid secretion in humans. Gastrin does not have a role in basal acid secretion in normal individuals.
Parietal cell activation and the resultant acid secretory response is greater to a combination of agonists than the sum of the responses to the agents used singly. This increase in responsiveness is termed potentiation. Potentiating interactions are most apparent when the stimulants use different second messenger systems, for example, acetylcholine and histamine. Conversely, blockade of receptors to one stimulant also blocks responsiveness to the other agonist. Because of this interaction, blockade of histamine receptors by agents like cimetidine decreases responsiveness to acetylcholine. Blockage of acetylcholine release by vagotomy decreases responsiveness to histamine secreted by gastric mast cells.
Both vagotomy and histamine2 receptor antagonists decrease basal acid secretion by approximately 80%.


89. As a meal is emptied from the stomach, gastric acid secretion gradually returns to baseline. Which of the following statements correctly characterize control of gastric acid secretion?

a. In humans, the most important inhibitory influence on gastrin release is exposure of the gastric mucosa to luminal acid
b. Acidification of the antral lumen causes reciprocal increases in somatostatin release and decreases in gastrin secretion
c. Antral distension stimulates gastric acid secretion
d. Acidification of the duodenal bulb inhibits gastric acid secretion
e. Exposure of the duodenum to hyperosmolar solutions inhibits acid secretion
Answer: a, b, d, e

Inhibitory regulation of gastric acid secretion is accomplished by central nervous system, gastric and small intestinal mechanisms. In humans, the most clearly established gastric inhibitory influence is suppression of gastrin release by exposure of the antral mucosa to luminal acid. Antral acidification causes release of gastric mucosal somatostatin which is linked reciprocally to decreases in gastrin secretion. Antral distension inhibits gastric acid secretion.
The inhibitory phase of gastric acid secretion begins with entry of the products of digestion into the proximal duodenum. Acidification of the duodenal bulb and exposure of the duodenum to hyperosmolar solutions and those containing fat potently inhibit acid secretion.


90. Which of the following statements correctly characterizes gastric motor activity associated with ingestion of a meal?

a. Ingested gastric volumes are accommodated with little increase in pressure by reflex relaxation of the proximal stomach
b. Receptive gastric accommodation is unaffected by proximal gastric vagotomy
c. In humans, liquid emptying occurs more quickly than solid emptying
d. Gastric emptying of liquids is not affected by proximal gastric vagotomy
Answer: a, c

With ingestion of a meal, increasing gastric volumes are accommodated with little increase in intragastric pressure by relaxation of the proximal stomach. This process, termed receptive relaxation, is mediated by a reflex carried by the vagal nerve. After the meal has been ingested, the proximal stomach is the predominant determinant of the rate of gastric emptying of liquids due to the gastroduodenal pressure gradient generated by proximal gastric contractions. Liquid emptying occurs more rapidly than emptying of solids, in part, because liquids are not subject to the sieving actions of the pylorus.
Truncal and proximal gastric vagotomy abolish receptive relaxation. Following vagotomy, an increased gastroduodenal pressure gradient is observed and correlates with accelerated liquid emptying. Emptying of solids is usually not significantly altered by proximal gastric vagotomy.


91. It is widely agreed that the gastric mucosa secretes bicarbonate in addition to acid. Gastric secretion of bicarbonate is correctly characterized by which of the following statements?

a. Bicarbonate is secreted by chief cells within gastric crypts
b. Gastric bicarbonate secretion is stimulated by acetylcholine
c. Gastric bicarbonate secretion during fasting results in luminal pH above 6 in normal individuals
d. Prostaglandin E2 is a potent stimulant of gastric bicarbonate secretion
Answer: b, d

The gastric cells responsible for bicarbonate secretion are believed to be surface mucous cells facing the gastric lumen between crypts. Although the total amount of gastric bicarbonate secreted is only a small fraction of total acid secretion, pH close to neutrality is maintained near the mucosal surface while bulk luminal pH is highly acidic. Cholinergic agonists, vagal stimulation, and sham feeding all increase gastric bicarbonate secretion. Prostaglandin E2 and its synthetic derivatives are potent stimulants of bicarbonate secretion as well. Conversely, indomethacin and other drugs that inhibit prostaglandin formation decrease mucosal bicarbonate secretion.


92. Gastric mucosal blood flow is regulated by neural, hormonal, and locally active influences. Which of the following statements correctly characterizes gastric blood flow?

a. Stimulation of sympathetic nerves supplying the stomach is followed by gastric mucosal hyperemia and increased total gastric blood flow
b. Vagal nerve stimulation is accompanied by decreased gastric mucosal blood flow
c. Stimulants that increase acid secretion increase mucosal blood flow
d. In humans, prostaglandins increase mucosal blood flow at doses that inhibit gastric acid secretion
Answer: c, d

Because the gastric mucosa is metabolically highly active, control of gastric mucosal blood flow is of great physiologic significance. Almost all stimuli that increase acid secretion also increase gastric blood flow. A large number of gastrointestinal hormones stimulate gastric blood flow, most because of their ability to increase acid secretion. Thus, gastrin is a potent stimulant of blood flow, in proportion to its ability to increase acid secretion. Vagal nerve stimulation has the net effect of increasing mucosal and total gastric blood flow; sympathetic nerve stimulation is accompanied by opposite effects. Prostaglandins are important endogenous regulators of gastric blood flow. Prostaglandins of the E class increase blood flow at doses that suppress acid secretion. Inhibition of cyclo-oxygenase activity by indomethacin causes a reduction in resting gastric blood flow.


93. Which of the following statements regarding intrinsic factor is/are correct?

a. Intrinsic factor is produced in chief cells located in the gastric fundus
b. Total gastrectomy is followed by folate deficiency due to vitamin malabsorption secondary to intrinsic factor deficiency
c. Intrinsic factor secretion, like that of acid, is stimulated by gastrin, histamine, and acetylcholine
d. Intrinsic factor deficiency accompanies H pylori-caused antral gastritis
Answer: c

The gastric mucosa is the site of production of intrinsic factor, which is a necessary co-factor for the absorption of vitamin B12 by the ileal mucosa. Total gastrectomy and atrophic gastritis involving the proximal oxyntic mucosa are regularly followed by vitamin B12 deficiency, manifest as pernicious anemia. Acid-secreting parietal cells are the site of intrinsic factor synthesis. Like acid secretion, intrinsic factor secretion is stimulated by gastrin, histamine, and acetylcholine


94. A 24-year-old woman develops epigastric pain and has a diagnosis of duodenal ulcer confirmed by esophagogastroduodenoscopy. The patient is in the third month of a pregnancy. The most appropriate treatment would be:

a. Proximal gastric vagotomy
b. Misoprostol 400 mg b.i.d.
c. Sucralfate 1 gm q.i.d.
d. Cimetidine 400 mg b.i.d.
Answer: c

Cimetidine, ranitidine, famotidine and newer H2 receptor antagonists bind competitively to parietal cell histamine receptors to produce reversible inhibition of acid secretion. While the pharmacokinetic profiles of the H2 receptor antagonists differ, when administered at equipotent doses, the agents produce similar degrees of acid suppression and similar rates of ulcer healing. Cimetidine interacts with the hepatic microsomal enzyme system and may increase blood levels and pharmacological effects of drugs that depend upon hepatic metabolism.
Misoprostol, a derivative of prostaglandin E, causes endoscopic healing in 60% of patients at 4 weeks. The major side effect of misoprostol is diarrhea due to effects upon gut smooth muscle contractility. Uterine bleeding has been reported in some women using the drug and the agent has potential abortifacient actions. For this reason, misoprostol is contraindicated in pregnancy.
Sucralfate, the aluminum salt of sulfated sucrose, has virtually no systemic absorption, and for this reason, is the drug of choice for pregnant patients. When administered at a dose of 1 gm four times daily, 80% of ulcers will heal by 6 weeks.


95. Helicoabacter pylori has been investigated as a possible etiologic agent in duodenal ulceration. Which of the following statement(s) regarding H pylori infection in humans is/are correct?

a. H pylori may be isolated from antral gastric mucosa in nearly 100% of patients with active duodenal ulceration but only 1–2% of normal volunteers
b. H pylori possess cell surface receptors that bind to small intestinal mucous cells
c. Therapeutic regimens for duodenal ulcer that eliminate the organism are associated with lower ulcer recurrence rates than those in which the organism persists
d. The incidence of the organism in the normal population increases with age
e. Antral gastritis is associated with development of duodenal ulcer
Answer: c, d, e

Helicobacter pylori has received enormous investigative attention in recent years as a possible infectious cause of peptic ulceration. The evidence that H pylori causes ulcers is substantial but largely inferential. Antral gastritis is nearly always present in patients with duodenal ulceration. H pylori infestation of antral mucosa is believed to cause gastritis. While normal small intestinal cells do not permit H pylori binding, areas of gastric metaplasia are usually demonstrated in the duodenal mucosa immediately surrounding the ulcer. Resolution of gastritis follows eradication of the organism and drug regimens that are bactericidal are associated with lower rates of ulcer recurrence than those that have no anti-bacterial actions.
However, it is clear that not all patients infested with H pylori develop ulceration. Half of patients with dyspepsia but no ulceration have evidence of H pylori infestation and 20% of healthy volunteers can be demonstrated to harbor the organism. The incidence of infestation increases with age in the asymptomatic population.


96. A 40-year-old male undergoes treatment of acute duodenal ulceration with cimetidine 400 mg b.i.d. and has resolution of symptoms by 6 weeks. The medication is continued as a nocturnal maintenance dose at the end of a three month treatment course. Recurrent symptoms develop 6 months after initial diagnosis and repeated endoscopic examination reveals recurrent ulceration. Biopsies of antral mucosa demonstrate moderate gastritis and the presence of H pylori. Medical management designed to eradicate H pylori and heal ulceration should include which of the following agents?

a. Cimetidine
b. Bismuth subcitrate
c. Amoxicillin
d. Metronidazole
e. Vancomycin
Answer: a, b, c, d

The observation that H pylori infection has an important role in ulcer pathogenesis has led to development of antimicrobial therapy for ulceration. Most successful regimens are based on a bismuth compound (colloidal bismuth subsalicylate or colloidal bismuth subcitrate) plus metronidazole, alone or in combination with amoxicillin or tetracycline. Bismuth compounds act locally and achieve gastric concentrations above the MIC for 90% of H pylori isolates. Metronidazole is secreted into the stomach at high concentration, and the in vivo activity of metronidazole is not diminished by gastric acidity. Triple therapy with bismuth, metronidazole, and tetracycline or amoxicillin eradicates H pylori in 90% of cases, compared to 0% eradication with ranitidine. Inclusion of an H2 receptor antagonist or omeprazole has been reported to increase efficacy of antimicrobial therapy. Currently, antimicrobial therapy has been recommended for peptic ulcer disease resistant to conventional therapy, including patients with ulcer relapse while on maintenance therapy and failure to heal in spite of H2 receptor antagonist or omeprazole therapy.


97. Development of duodenal ulceration is dependent upon gastric acid secretion. Which of the following statements correctly characterizes acid secretion in duodenal ulcer patients?

a. Groups of duodenal ulcer patients demonstrate decreased basal acid secretion
b. Maximal acid output to histamine averages 40 mEq/h in duodenal ulcer patients, twice that of normal
c. Tissue gastrin levels, on average, are twice normal in patients with active ulceration
d. Exogenously administered somatostatin is ineffective in suppressing acid secretion in patients with active ulceration
Answer: b

The formation of duodenal ulcer is dependent on gastric secretion of acid and pepsin. As a group, duodenal ulcer patients have an increased capacity for gastric acid secretion relative to normal individuals, manifest by increased basal acid secretion, increased acid response to meal ingestion, and increased responsiveness to histamine stimulation. No definite evidence links abnormalities in gastrointestinal hormone secretion with increased acid secretion. Tissue gastrin content and circulating levels of gastrin are normal (excluding Zollinger-Ellison syndrome patients). Secretion of endogenous somatostatin and responsiveness to exogenously administered somatostatin are likewise normal.


98. A 45-year-old man undergoes proximal gastric vagotomy for treatment of intractable duodenal ulceration. What physiologic alterations might be anticipated as a consequence of the operation?

a. Reduction of basal acid secretion by approximately 25%
b. Accelerated gastric emptying of liquids
c. Accelerated gastric emptying of solids
d. Fasting hypergastrinemia
e. Postprandial hyperinsulinemia
Answer: b, d

Division of cholinergic vagal fibers directly affects parietal cell acid secretion by reducing stimulatory input. Basal acid secretion is diminished by approximately 80% and maximal acid output in response to pentagastrin stimulation is reduced by about 70%. Fasting hypergastrinemia and an exaggerated gastrin response to meal ingestion is observed due to loss of feedback inhibition of gastrin release and gastrin cell hyperplasia. Release of pancreatic polypeptide, secretin and cholecystokinin may be decreased.
Proximal gastric vagotomy accelerates gastric emptying of liquids due to a loss of receptive relaxation. In contrast, gastric emptying of solids is usually not affected by proximal gastric vagotomy.


99. Which of the following statements regarding postoperative rates of recurrent ulcer and dumping is/are correct?

a. Truncal vagotomy and antrectomy is associated with persistent dumping in 10–15% of patients
b. Recurrent ulceration following truncal vagotomy and pyloroplasty is observed in 25% of patients within 10 years of operation
c. Patients that undergo proximal gastric vagotomy have a risk of recurrent ulcer of 10–15% and a risk of persistent dumping approximating 1%
d. Recurrent ulceration occurs in 5% of patients that undergo truncal vagotomy and antrectomy
Answer: a, c

Surgical recommendations for treatment of peptic ulceration should be based upon safety, freedom from long-term postoperative symptoms, and avoidance of recurrent ulceration. Proximal gastric vagotomy has an operative mortality of less than 1% and a risk of persistent dumping symptoms of approximately 1%. The low incidence of postoperative symptoms is associated with a relatively high risk of recurrent ulceration, estimated to be 10% to 15% at 5 years postoperatively. After truncal vagotomy and pyloroplasty, dumping is initially present in 10%, and is persistent or severe in 1%. Recurrent ulceration is observed in 10% of patients that undergo truncal vagotomy and pyloroplasty. Truncal vagotomy and antrectomy is associated with the lowest risk of recurrent ulceration, 1–2%, but the greatest incidence of postoperative dumping symptoms, 10–15%.


100. Which of the following statement(s) is/are correct with regard to pyloric obstruction secondary to peptic ulceration?

a. Pyloric obstruction is suggested by hypochloremic hyponatremic alkalosis
b. Pyloric obstruction is suggested by hypochloremic hypokalemic alkalosis
c. Approximately 80% of patients with benign gastric outlet obstruction obtain permanent relief of symptoms by endoscopically-directed balloon dilatation
d. The lifetime risk of pyloric obstruction in peptic ulcer patients is 40%
Answer: b

Repeated episodes of ulceration and healing can lead to scarring and pyloric stenosis. The lifetime risk of this complication approximates 10%. Gastric outlet obstruction is characterized by the development of hypochloremic hypokalemic alkalosis due to loss of HCl through vomiting and renal compensatory mechanisms that conserve H+ at the expense of secreted K+. Although 85% of pyloric stenoses are technically amenable to balloon dilatation, fewer than 1 in 3 will achieve permanent relief of symptoms through this means.


101. A 42-year-old man with a recently diagnosed duodenal ulcer develops melena and near-syncope. After fluid resuscitation, upper gastrointestinal endoscopy is performed. During the examination, a 1 cm ulcer is noted in the proximal duodenum. A fresh clot is observed within the ulcer and blood is noted to be oozing around the clot. Optimal therapy would consist of which of the following?

a. Angiographic embolization of the gastroduodenal artery
b. Irrigation of the clot followed by endoscopic application of a heat probe
c. Transfusion and intravenous cimetidine
d. Angiographic infusion of vasopressin into the gastroduodenal artery
e. Transfusion and oral omeprazole
Answer: b

The ability to visualize bleeding duodenal ulcers endoscopically has led to attempts to treat hemorrhage endoscopically. Thermal coagulation may be achieved by bipolar electrocoagulation or by direct application of heat through a probe. An NIH Consensus Development Conference has recommended endoscopic hemostatic therapy in selected patients. Hematemesis, age over 60, and serious medical co-morbidity are clinical features that mandate endoscopic therapy. Rebleeding during hospitalization and the endoscopic findings of visible vessel, oozing, or bleeding associated with an adherent clot are other indications for endoscopic hemostasis. Operative intervention is appropriate for massive hemorrhage leading to shock or cardiovascular instability, prolonged blood loss requiring continuing transfusion, recurrent bleeding during medical therapy or after endoscopic therapy, and recurrent hemorrhage requiring hospitalization. Operative therapy should consist of duodenotomy with direct ligation of the bleeding vessel within the ulcer base followed by a procedure to permanently reduce acid production.


102. A 50-year-old patient has undergone truncal vagotomy and antrectomy with Billroth II reconstruction two years ago. The patient now complains of recurrent postprandial pain, nausea, and vomiting. Endoscopic examination reveals bile in the stomach; endoscopic biopsies demonstrate histologic evidence of moderately severe gastritis. No other endoscopic abnormalities are noted. Appropriate therapy could include:

a. Octreotide administration
b. Total gastrectomy
c. Conversion of Billroth II gastrojejunostomy to Billroth I gastroduodenostomy
d. Conversion of Billroth II gastrojejunostomy to Roux-en-Y gastrojejunostomy
e. Roux-en-Y hepaticojejunostomy
Answer: d

Symptoms related bile reflux gastritis occur transiently in 10% to 20% of patients after truncal vagotomy and resection or drainage. Symptoms persist in only 1% to 2%. No completely satisfactory solution to bile reflux gastritis exists. Medicinal and dietary treatments have not been proven to be beneficial. Operative diversion of biliary secretions away from the gastric mucosa by construction of a Roux-en-Y gastrojejunostomy with an intestinal limb of 50 to 60 cm has been widely reported. The procedure eliminates bilious vomiting in nearly 100% of patients, but pain persists in up to 30%, and 20% develop delayed gastric emptying as a result of the procedure.


103. A 50-year-old male with a 2 year history of duodenal ulceration develops sudden, severe epigastric pain 4 hours prior to evaluation. Physical examination reveals T 101° F, pulse 80, BP 125/90, diminished bowel sounds, and abdominal muscular rigidity. An upright chest x-ray reveals pneumoperitoneum. At laparotomy, an anterior perforation in the first portion of the duodenum is observed. Optimal treatment would include:

a. Omental patch of the perforation followed by truncal vagotomy and antrectomy after 8 weeks
b. Omental patch of the perforation followed by truncal vagotomy and pyloroplasty after 8 weeks
c. Omental patch of the perforation followed by chronic cimetidine administration
d. Omental patch of the perforation plus proximal gastric vagotomy
e. Omental patch of the perforation only
Answer: d

Simple omental patching of a perforation in patients with chronic ulcer disease does not yield satisfactory long-term results. Up to 80% of patients so treated develop recurrent ulceration and 10% develop secondary complications. A definitive ulcer operation should be performed during the initial laparotomy if the following circumstances apply: 1) there has been no preoperative shock, 2) the perforation has been present for less than 48 hours, and 3) no life-threatening medical co-morbidity exists. Omental patching of the perforation combined with proximal gastric vagotomy is a preferred approach because it combines safety, freedom from disabling postoperative symptoms, and a low rate of recurrent ulceration.


104. Which of the following clinical circumstances have been identified as predisposing factors for the development of stress ulceration?

a. Intraperitoneal sepsis
b. Hemorrhagic shock
c. Isolated tibial fracture
d. 50% total surface area second degree burn
e. Adult respiratory distress syndrome
Answer: a, b, d, e

Several risk factors or predisposing clinical conditions have been identified for stress ulceration. Specific risk factors include adult respiratory distress syndrome, multiple trauma, major burn of over 35% of body surface area, oliguric renal failure, large transfusion requirements, hepatic dysfunction, hypotension, prolonged surgical procedures, and sepsis from any source. A direct correlation has been shown between acute upper gastrointestinal hemorrhage and the severity of critical illness.


105. Type I gastric ulcers are located in the gastric body, usually along the lesser curvature. Which of the following statements correctly characterize type I gastric ulcers?

a. Normal to low acid secretion
b. Associated duodenal ulceration
c. High frequency of blood group A
d. Associated hypergastrinemia frequent
Answer: a, c

Gastric ulcers are divided into categories based on their location and gastric acid secretory status. A type I gastric ulcer is an ulcer in the body of the stomach, usually along the lesser curvature, associated with large volumes of secretion with a low to normal acid output. Type I ulcers are not associated with duodenal, pyloric, or prepyloric mucosal abnormalities. There is a slight predominance of patients with blood group A in this type of gastric ulcer.
Type II gastric ulcer is located in the body of the stomach in combination with a duodenal ulcer. These patients are usually acid hypersecretors. About 23% to 25% of gastric ulcers are type II. A type III gastric ulcer is characterized as a prepyloric ulcer and accounts for about 23% of lesions. Patients with this lesion are typically acid hypersecretors.
Type IV gastric ulcer occur high on the lesser curvature near the gastroesophageal junction. In the United States, the incidence of type IV gastric ulcer is less than 10%.


106. Which of the following statement(s) is/are correct regarding gastric ulcers greater than 3 cm in size?

a. Giant gastric ulcers occur in 30–40% of cases along the greater curvature
b. The risk of malignancy increases with increasing size of the ulcer
c. The treatment of choice for giant gastric ulcer is resection to include the ulcer
d. Giant gastric ulcer is a complication of intraarterial hepatic chemotherapy
Answer: b, c

A giant gastric ulcer is defined as an ulcer whose diameter is 3 cm or greater. The lesser curvature is the most common site, with only 3% to 4% occurring along the greater curvature. Gastric ulcers often penetrate into contiguous structures such as spleen, pancreas, liver, and transverse colon, and can be falsely diagnosed as a nonresectable malignancy, despite normal biopsy results. Most reports cite an incidence of malignancy ranging from 6% to 30%, increasing with the size of the ulcer. Because of the high likelihood of complications from giant gastric ulcer, early operation is the treatment of choice. The operation of choice is resection, including the ulcer. If the ulcer has penetrated adjacent structures and cannot be dissected free, the stomach wall can be incised circumferentially, leaving the ulcer intact and behind, and the gastric resection completed.


107. With regard to benign gastric ulceration, the most common location of disease is which of the following?

a. Along the greater curvature
b. Immediately distal to the esophagogastric junction along the lesser curvature
c. In the area of the incisura angularis along the lesser curvature
d. Within the gastric antrum
Answer: c

Gastric ulcers can occur anywhere in the stomach, although they usually present on the lesser curvature near the incisura angularis. About 60% are located at or slightly above the angularis. Fifteen percent to 23% of gastric ulcers are within the gastric antrum and 10% are high on the lesser curvature. Only 5% of gastric ulcers are found on the greater curvature. In addition, 97% of all gastric ulcers occur within 2 cm of the junctional zone between fundic and antral mucosa. Gastric ulcers appear at different distances from the pyloric sphincter because the antrum extends for variable (2 to 16 cm) distances from the pylorus. It is interesting that with increasing age this junctional zone moves proximally along the lesser curvature, as does the incidence of gastric ulcer.


108. Which of the following statement(s) regarding gastric mucosal defense is/are correct?

a. Gastric mucus, produced by the surface epithelial cells, forms an unstirred layer over the gastric surface
b. Gastric parietal cells produce a bicarbonate-rich fluid
c. Production of gastric bicarbonate is stimulated by prostaglandins and inhibited by non-steroidal antiinflammatory drugs (NSAIDs)
d. Gastric mucus provides substantial buffering capacity that maintains near-neutrality near the epithelial surface
Answer: a, c

Gastric mucus is produced by the surface epithelial cell. In addition to serving as a lubricant, mucus also enhances mucosal defense by forming an unstirred layer overlying the epithelial surface. In humans, the thickness of this gel layer has been reported to be about 500 µm. The vacuoles containing mucus in the cytosol of the surface epithelial cell are released by cholinergic stimulation and prostaglandins. Synthesis and release are inhibited by aspirin-like compounds that inhibit cyclooxygenase. Gel mucus retards proton mobility by a factor of only three or four, a degree that is inadequate to maintain a near neutral pH at the apical membrane of the surface epithelial cell.
In addition to producing mucus, the gastric surface epithelial cells secrete a bicarbonate-rich fluid. The amount of bicarbonate secreted is about 5% to 10% of the amount of acid that the same surface of mucosa can produce. As with mucus, bicarbonate secretion is stimulated by cholinergic agents and prostaglandins and inhibited by cyclooxygenase inhibitors.
The release of bicarbonate into the gel mucus provides a significant mucosal defense by maintaining a near neutral acid-base milieu at the apical membrane of the surface epithelial cells. When the luminal pH is around 3.0, the apical membrane of the surface epithelial cell may be exposed to a pH of about 5.0.


109. A 35-year-old smoker is involved in a house fire and receives a 45% total surface area burn. One half of the burned surface appears to be third degree. On the third post-burn day, the patient is noted to have bloody drainage from a nasogastric tube and a decrease of 5% in his hematocrit. Appropriate management should include which of the following?

a. Urgent upper gastrointestinal contrast study to delineate site of bleeding
b. Immediate selective arteriography via the left gastric artery to diagnose and treat presumed stress ulceration
c. Urgent esophagogastroduodenoscopy to diagnosis the cause of bleeding
d. Urgent intravenous infusion of vasopressin at 0.2–0.4 IU/min
Answer: c

Patients who have sustained a major thermal burn of 35% or more of their body surface area are at a predictably high risk for the development of gastric erosions and hemorrhage. Endoscopy has demonstrated that gastric erosions are present in 93% of these patients, whereas the occurrence of severe acute upper gastrointestinal hemorrhage in severely burned patients ranges between 25% to 50%.
At least 60% of patients at risk develop stress erosions within 1 to 2 days after the precipitating event. Painless upper gastrointestinal bleeding may be the only clinical sign. The onset of hemorrhage is often delayed, usually occurring 3 to 10 days after the onset of the primary disease.
Esophagogastroduodenoscopy is the diagnostic modality of choice to confirm the diagnosis and to differentiate stress erosion from other sources of upper gastrointestinal hemorrhage. Correct identification of the bleeding source is made in greater than 90% of instances. If endoscopy is not diagnostic, visceral angiography through selective catheterization of the left gastric or splenic vessels may provide information regarding the primary vessel supplying the bleeding site. In contrast, barium examinations are usually of little value, due to the superficial nature of stress erosions, and in fact may be detrimental by interfering with the interpretation of subsequent arteriography.


110. Agents demonstrated to have an efficacy of greater than 90% for prophylactic treatment of stress ulceration include which of the following?

a. Antacids
b. H2 receptor antagonists
c. Sucralfate
d. Misoprostil
Answer: a, b, c

The hourly administration of antacid (30 to 60 mL) by nasogastric tube, maintaining the gastric luminal fluid at pH above 3.5, has proven to be effective prophylaxis. In a study of 100 seriously ill patients who were randomly assigned to receive placebo or antacid prophylaxis, bleeding was detected in 25% of patients given no prophylaxis, compared to 4% of patients given antacids through the nasogastric tube. In a review of data derived from 16 prospective trials, when overt bleeding manifested by melena, hematemesis, or transfusion requirement was used as the minimum criterion, there was no significant difference in risk of bleeding, comparing antacids and cimetidine.
Continuous infusions of any of the H2-receptor antagonists provides more consistent maintenance of an intraluminal gastric pH of greater than 3.5 than do the standard intermittent-infusion regimens. Advantages of continuous infusion of these agents include a potential reduction in toxicity, decreased pharmacy costs and nursing duties, and possible enhancement of therapeutic benefit. Controlled trials suggest that sucralfate, 1 g every 6 hours, may be as effective as antacids or cimetidine prophylactically. In 100 critically ill patients, bleeding occurred in 6% of patients receiving antacids or cimetidine, while none of the 34 patients on sucralfate bled.
Given exogenously, natural or synthetic prostaglandins of the E, F, and I series inhibit gastric acid secretion. One group compared the efficacy of 15(R)-15 dimethyl PGE2 given at antisecretory doses to antacids and found that stress-related bleeding occurred in 50% of patients given the synthetic prostaglandin derivative, compared to only 14% of patients receiving antacids.


111. A 45-year-old male presents with symptoms of epigastric pain, worsened with ingestion of food. Physical examination is normal. Upper abdominal ultrasonography is unremarkable. Contrast radiography reveals a 2 cm ulcer in the gastric fundus along the lesser curvature. Therapy with omeprazole 20 mg per day is begun but symptoms persist 3 weeks later. Appropriate management includes which of the following?

a. Increase in omeprazole dose to 40 mg per day
b. Addition of sucralfate 1 gm every 8 hours
c. Addition of cimetidine 200 mg b.i.d.
d. Esophagogastroduodenoscopy with biopsy of ulceration
Answer: d

About 5% of ulcers appearing radiographically benign are malignant} Gastroscopy is the most reliable method of distinguishing benign and malignant gastric ulcer, with an accuracy of more than 97% if multiple biopsies and brushings for cytology are performed. Clinical features prompting early endoscopic evaluation include major weight loss, symptoms of gastric outlet obstruction, a palpable abdominal mass, and stool hemoccult positivity or blood loss anemia. Endoscopic features that suggest malignancy include an exophytic mass, abnormal or disrupted mucosal folds, necrotic ulcer crater, bleeding from the edge of the ulcer crater, a stepwise depression of the ulcer edge, heaped-up margins, or small extensions of the ulcer that blur a portion of the ulcer wall. If initial biopsies do not demonstrate malignant cells but the endoscopic appearance strongly suggests that underlying the ulcer is a carcinoma, repeat endoscopy with deeper biopsies should be undertaken.


112. A 52-year-old woman is hospitalized with acute upper gastrointestinal hemorrhage. Endoscopic examination reveal a 2.5 cm ulcer in the area of the incisura angularis. The remainder of the endoscopic examination is normal. Continued bleeding requires operative therapy. Optimal therapy consists of which of the following?

a. Gastrotomy with oversewing of the bleeding site
b. Distal gastrectomy including the area of ulceration
c. Proximal gastric vagotomy and oversewing of the bleeding ulcer
d. Truncal vagotomy, pyloroplasty, and oversewing of the bleeding ulcer
Answer: b

A distinction should be made among the different types of gastric ulcer in selecting the most appropriate operative procedure, because treatment varies according to location, coexistent duodenal ulcer disease, and acid secretory status. The elective operation of choice for a type I benign gastric ulcer is a distal gastrectomy with gastroduodenal anastomosis. Gastrojejunostomy is an acceptable alternative. The ulcer should be included in the antrectomy specimen. The operative mortality rate associated with this procedure is 2% to 3%, the recurrence rate is 3%, and a good to excellent clinical result can be anticipated in more than 90% of patients. The addition of truncal vagotomy does not appear to diminish the recurrence rate.
Definitive treatment for hemorrhage is accomplished by a procedure designed to control bleeding in addition to preventing recurrent ulceration. An antrectomy, which includes the ulcer with gastroduodenostomy, is considered the procedure of choice for surgical treatment of this complication. The quoted operative mortality rates in this setting range from 10% to 40%.


113. A 25-year-old man is involved in an automobile accident with resultant injuries including bilateral closed femur fractures, left pulmonary contusion, and closed head injury. On post-injury day 4, significant upper gastrointestinal hemorrhage begins. Endoscopic examination reveals an area of confluent ulceration with bleeding in the gastric fundus. Endoscopic hemostasis fails. Appropriate immediate management includes:

a. Lavage of gastric contents with iced saline
b. Urgent total gastrectomy
c. Selective arterial infusion of vasopressin via the left gastric artery
d. Insertion of Sangstaken-Blakemore balloon
Answer: c

Initial efforts to control gastric hemorrhage consist of gastric lavage using warmed saline. Lavage serves to fragment existing clots and to remove any pooled blood, reducing fibrinolysis at bleeding sites. Over 80% of patients who present with upper gastrointestinal hemorrhage stop bleeding using this approach. Definitive treatment of ongoing acute active stress bleeding by antacids is largely unsuccessful. Administration of H2-receptor blocking agents once active gastrointestinal bleeding has commenced is also usually ineffective as a definitive form of therapy.
The endoscope has become the preferred therapeutic as well as diagnostic instrument with electrocautery and laser photocoagulation capabilities. If endoscopic therapy fails, angiography offers an additional means for the control of bleeding by selective infusion of vasopressin into the splanchnic circulation via the left gastric artery. Vasopressin is administered by continuous infusion through the catheter at a rate of 0.2 to 0.4 IU/min for a maximum of 48 to 72 hours.
About 10% to 20% of patients with acute stress ulcers continue to bleed or have recurrent bleeding despite these measures. In these patients, total gastrectomy has a mortality ranging from 17% to 100%. In general, operative mortality rates for acute stress-induced hemorrhage range from 30% to 60% regardless of the surgical procedure undertaken.

114. Which of the following statements regarding gastroplasty and gastric bypass for morbid obesity is/are correct?

a. Horizontal gastroplasty techniques that rely on a single horizontal application of a stapling device are associated with weight loss “failure” rates of 40% to 70%
b. Gastric bypass is followed by progressive weight loss over a period of 36 months
c. Gastric bypass is associated with a “failure” of weight loss in 10 to 15% of patients
d. With three superimposed applications of a stapling device, gastric bypass staple line dehiscence occurs in less than 2%
Answer: a, c, d

Horizontal gastroplasties include a single application of a 90-mm stapling device without suture reinforcement of the “stoma’’ between upper and lower gastric pouches or a double application of staples with either a central or lateral prolene-reinforced stoma. The failure rates (loss of less than 40% excess weight) for horizontal gastroplasty procedures ranges from 40% to 70%. The vertical banded gastroplasty (VBGP) is a procedure in which a stapled opening is made in the stomach with the stapling device 5 cm from the cardioesophageal junction. Two applications of a 90-mm stapling device are made between this opening and the angle of His, and a 1.5 5 cm strip of polypropylene mesh is wrapped around the stoma on the lesser curvature and sutured to itself.
Gastric bypass can be performed with placement of staples in a vertical or horizontal direction; the vertical direction is preferred because there is less risk of gastric pouch devascularization or splenic injury. With three superimposed applications of a 90-mm stapler, the incidence of staple line disruption has been less than 2%.
Roux-en-Y gastric bypass has significantly better weight loss than VBGP. Although 10% to 15% of patients fail gastric bypass, weight loss seems to remain stable in most patients over 5 years or more after surgery.


115. Severe obesity is associated with a large number of associated problems that form the basis of the term morbid obesity. Documented causes of excess mortality in severely obese patients include:

a. Coronary artery disease
b. Hypertension
c. Adult-onset diabetes mellitus
d. Obesity hypoventilation and sleep apnea
e. Pulmonary embolization
Answer: a, b, c, d, e

Morbid obesity is arbitrarily defined as 100 pounds above ideal body weight as defined by actuarial tables. Premature death is much more common in the severely obese; morbidly obese men aged 25 to 34 years have a 12-fold increase in mortality relative to normal. Causes of early mortality include coronary artery disease, hypertension, impaired ventricular function, diabetes mellitus, sleep apnea and other hypoventilation syndromes, pulmonary embolization, and necrotizing soft tissue infections.


116. Jejunoileal bypass was formerly performed as a weight reduction procedure. The operation has now been abandoned due to the development of serious long-term complications associated with the procedure. Which of the following statements correctly characterize results following jejunoileal bypass?

a. Kidney stones occur with increased frequency due to increased absorption of pyruvate from the colon
b. The most serious complication of jejunoileal bypass is development of cirrhosis due to protein calorie malnutrition
c. Bacterial overgrowth in the bypassed segment can be treated with oral vancomycin
d. Rapid weight loss following jejunoileal bypass is associated with development of gallstones
Answer: b, d

Jejunoileal bypass is associated with a number of early and late complications. Malabsorption of bile salts, coupled with rapid weight loss, significantly increases risk of gallstone development. Multiple kidney stones result from excessive absorption of oxylate from the colon where oxylate is ordinarily chelated with calcium. Malabsorption results in severe diarrhea, electrolyte abnormalities, metabolic acidosis and anemia. Bacterial overgrowth in the bypassed intestinal segment coupled with protein malabsorption is postulated to be responsible for development of cirrhosis, the most serious complication of jejunoileal bypass. Bacterial overgrowth can be temporarily suppressed by metronidazole. Development of hepatic dysfunction is an indication for reversal of the bypass.


117. Which of the following statements is correct with regard to gastric bypass for obesity?

a. Rapid weight loss following successful gastric bypass for obesity is associated with an increased risk of developing cholelithiasis
b. Marginal ulcer develops in 25% of gastric bypass patients
c. Vitamin B12 deficiency is a potential complication of gastric bypass due to gastric mucosal atrophy
d. Anastamotic leak after gastric bypass is often heralded by bradycardia
Answer: a

The most serious complication after gastric bypass for obesity is anastamotic dehiscence. Leak is presumed to occur secondary to gastric necrosis due to ischemia from staple line application or short gastric vessel ligation. Affected patients may have little pain, with tachycardia, tachypnea and fever as the only manifestations. Physical examination in morbidly obese patients with peritonitis is unreliable. Marginal ulcers occur in only 10% or less of gastric bypass patients and respond to H2 receptor antagonists. Rarely, polyneuropathy has been noted after gastric bypass, usually in association with intractable vomiting and protein calorie malnutrition. Vitamin B12 deficiency has been noted after gastric bypass due to decreased acid digestion of vitamin B12 in food; monthly B12 supplementation should be routine. Cholelithiasis occurs in about one third of morbidly obese patients and gallstone formation is accelerated in the early postoperative period by the effects of rapid weight loss.


118. With regard to operative treatment of gastric carcinoma, which of the following statements is/are correct?

a. Resectional margins of 2 cm are necessary to prevent recurrence due to intramural metastases
b. Prophylactic splenectomy has been shown to improve outcome in similarly staged patients
c. Extended lymphadenectomy including nodes along the aorta and esophagus has not been shown to improve survival in North American trials
d. Long-term survival is rare if adjacent organs must be resected to achieve local control
Answer: c, d

In gastric cancer, microscopic involvement of the resection margin by tumor cells is associated with poor prognosis. In contrast to colon cancer, gastric cancer frequently demonstrates extensive intramural spread. Retrospective studies suggest that a line of resection 6 cm from the tumor mass is necessary to ensure a low rate of anastamotic recurrence.
The value of extended lymphadenectomy in the treatment of gastric adenocarcinoma is controversial. The largest favorable experience has been reported by Japanese surgeons, where retrospective studies have suggested an improvement of approximately 10%, stage for stage, for patients with advanced disease. The benefits of extensive lymphadenectomy have not been confirmed in western countries.
Histologically positive lymph nodes are frequently present in the splenic hilum and along the splenic artery, and routine splenectomy has been practiced in some centers. Prophylactic splenectomy has not been demonstrated to improve outcome for similarly staged patients. Resection of adjacent organs may be required for local control if direct invasion has occurred. In this circumstance, operative morbidity is increased, and long-term survival is rare.


119. Which of the following statement(s) characterizing gastric lymphoma is/are correct?

a. More than half of gastrointestinal lymphomas occur in the stomach
b. The peak incidence of gastric lymphoma is in the 2nd and 3rd decade
c. Endoscopic biopsy is positive diagnostically in 90% of cases
d. Gastric perforation occurs in 40% of patients treated with cytolytic agents instead of gastrectomy
Answer: a, c

The stomach is the site of more than half of gastrointestinal lymphomas and is the most common organ involved in extranodal lymphomas. Gastric lymphoma is distinctly uncommon in children and young adults. The peak incidence is in the sixth and seventh decades. Radiologic findings are similar to those noted for adenocarcinoma. Endoscopic examination has become the diagnostic method of choice. Endoscopic biopsy, combined with endoscopic brush cytology, provides positive diagnosis in some 90% of cases. When gastric lymphoma is first diagnosed by endoscopic means, evidence of systemic disease should be sought. CT of chest and abdomen (to detect lymphadenopathy), lymphangiography, bone marrow biopsy, and biopsy of enlarged peripheral lymph nodes may all be appropriate.
A multimodality treatment program is used in most centers for primary gastric lymphomas, with gastrectomy as the first step in the therapeutic strategy. Increasing numbers of patients are treated with chemoradiation therapy alone. The risk of hemorrhage or perforation was frequently alluded to in the past and has probably been overstated. The risk of perforation in primary gastric lymphomas that are treated with cytolytic agents in unresected patients approximates 5%.


120. A patient with gastric adenocarcinoma undergoes subtotal gastrectomy. Pathological examination reveals that the tumor penetrates to the serosa. Regional lymph nodes are not involved. Distant metastases are not detected. What is the correct tumor stage and 5-year survival rate?

a. Stage I: 90% 5-year survival
b. Stage II: 45% 5-year survival
c. Stage III: 15% 5-year survival
d. Stage II: 15% 5-year survival
e. Stage III: 2% 5-year survival
Answer: b

For early lesions of the antrum or middle stomach, distal subtotal gastrectomy including 80% of the stomach provides satisfactory 5-year survival without increases in operative morbidity. Proximal gastric lesions or larger middle stomach lesions may require total gastrectomy or esophagogastrectomy to encompass the tumor. Regardless of the extent of gastric resection, patients with more advanced tumors fare poorly because of the increased likelihood of lymphatic and hematogenous spread.
The TNM system is shown in Table 25-2. Five-year survival for stage I patients (in situ carcinoma) is close to that of the normal population. In stage II, 5-year survival approximates 45%, while 15% of stage III patients survive 5 years. Prolonged survival with systemic metastases in negligible.


121. Which of the following conditions is considered to increase the risk of gastric cancer?

a. Pernicious anemia
b. Prior partial gastrectomy
c. Gastric hyperplastic polyps
d. Gastric adenomatous polyps
Answer: a, b, d

The risk of developing gastric cancer is greater in stomachs that harbor adenomatous polyps. The risk of developing cancer has been estimated at 10% to 20% and is greatest for polyps more than 2 cm in diameter. Hyperplastic polyps, while common in the normal population, do not have malignant potential. Gastric malignancy is increased in individuals with chronic gastritis associated with pernicious anemia. When pernicious anemia has been present for 5 years, the risk of gastric malignancy is twice that of age-matched controls. An increased risk of gastric carcinoids also exists in patients with pernicious anemia, presumably due to the effects of long-standing hypergastrinemia. A 3-fold increased risk of gastric cancer also exists in patients that have previously undergone partial gastric resection. Postgastrectomy cancer is a long-term concern with increased incidence of malignancy not observed until 15 years postoperatively.


122. A 55-year-old male is evaluated because of symptoms of epigastric pain and anorexia. Physical examination is normal except for guaiac positivity of stool. Upper endoscopic examination reveals a 1.5 cm ulcer along the lesser curvature of the stomach proximal to the incisura angularis. Optimal management consists of which of the following:

a. Sucralfate 1 gm q.i.d. for 8 weeks
b. Endoscopic biopsy of the ulcer rim
c. Endoscopic cautery of the ulcer base
d. Endoscopic biopsy of the ulcer base
e. Misoprostol 400 mg b.i.d. for 8 weeks
Answer: b

The symptoms produced by gastric cancer and benign gastric ulcer are non-specific and often similar. Pain is present in 70% of patients with gastric cancer and is usually constant, non-radiating, and not improved by food ingestion. Physical examination is usually normal in patients with early gastric cancer. Guaiac positivity is noted in one third.
Fiberoptic endoscopy is the definitive diagnostic method. Although the endoscopic appearance of gastric ulcers may suggest benign or malignant origins, definite distinction can only be made by gastric biopsy. Accurate diagnosis of gastric cancer can be made in 95% of cases if multiple biopsies are obtained from the ulcer rim. Biopsy of the ulcer base will more frequently reveal necrotic material.


123. Which of the following statement(s) regarding gastric leiomyosarcoma is/are correct?

a. Leiomyosarcomas occur with peak frequency in the 2nd and 3rd decades
b. The primary histological indicator of aggressive behavior is the number of mitoses per microscopic field
c. Leiomyosarcomas are usually radiosensitive
d. Lymphadectomy is not indicated during resection because metastases are usually hematogenous
Answer: b, d

Leiomyosarcomas occur with equal frequency in both sexes in the sixth and seventh decades of life. The tumor frequently may have prominent extraluminal growth and attain large size before causing symptoms. Leiomyosarcomas must be differentiated from their benign counterparts, leiomyomas. Grossly, the tumors are firm, gray-white masses; a pseudocapsule separating tumor from normal smooth muscle may occasionally be present. When the tumors reach a large size, central necrosis is common. Leiomyosarcomas are usually graded histologically, with the frequency of mitotic figures the prime indicator of aggressive behavior. Lesions with more than 5 to 10 mitoses per 10 high-power fields demonstrate increased metastasis.
Intraperitoneal sarcomatosis is frequent, as is local recurrence after resection. Metastasis occurs by way of the hematogenous route, and thus hepatic involvement is common. Lymphatic metastasis is observed in less than 10% of patients. Negative surgical margins must be ensured histologically, but lymphadenectomy is not indicated because of the low frequency of lymphatic metastasis. Leiomyosarcomas are not radiosensitive, and chemotherapy has not been shown to improve survival.

13 comments:

  1. Achnical reduces the stomach ache and manages the hunger of the patient. It also helps the esophagus to take the food safely to the stomach for proper digestion. It is a clinically tested product that has no side effects and helps the patient to manage the hunger easily. People prefer the herbal products more than other remedies because according to the facts herbal medicine has rarely any side effect. You can order the medicine from Herbal Care Products. Achalasia is basically the stomach and digestion issue that can be easily cured by the herbal medicine. It is better to start Achnical as the problem starts appearing.

    ReplyDelete
  2. There is convinced Natural Treatment for Achalasia that proposals a promising remedy for esophageal cancer.

    ReplyDelete
  3. Try Natural Treatment for Achalasia because the mixture of different natural ingredients makes a proper herbal remedy such as Achalasia Herbal Remedy produced by Natural Herbs Clinic. This herbal remedy work properly and can help to treat your achalasia successfully.

    ReplyDelete
  4. Great write-up, I am a big believer in commenting on blogs to inform the blog writers know that they’ve added something worthwhile to the world wide web!.. how to avoid bad breath naturally

    ReplyDelete
  5. HOW I GOT CURED OF HERPES VIRUS.

    Hello everyone out there, i am here to give my testimony about a herbalist called dr zubby. i was infected with herpes simplex virus 2 in 2013, i went to many hospitals for cure but there was no solution, so i was thinking on how i can get a solution out so that my body can be okay. one day i was in the pool side browsing and thinking of where i can get a solution. i go through many website were i saw so many testimonies about @dr_zubby4 on instagram on how he cured them. i did not believe but i decided to give him a try, i contacted him and he prepared the herpes for me which i received through fedex courier service. i took it for two weeks after then he instructed me to go for check up, after the test i was confirmed herpes negative. am so free and happy. so, if you have problem or you are infected with any disease kindly contact him on email via dr.zubbysolutionhome@gmail.com. or / whatssapp --+2348070673249
    This testimony serve as an expression of my gratitude. he also have
    herbal cure for, FEVER, DIARRHEA, FATIGUE, MUSCLE ACHES, LUPUS DISEASE, JOINT PAIN, POLIO DISEASE, PARKINSON'S DISEASE, ALZHEIMER'S DISEASE, CYSTIC FIBROSIS, SCHIZOPHRENIA, CORNEAL ULCER, EPILEPSY, FETAL ALCOHOL SPECTRUM, LICHEN PLANUS, COLD SORE, SHINGLES, CANCER, HEPATITIS A, B. DIABETES 1/2, HIV/AIDS, CHRONIC PANCERATIC, CHLAMYDIA, ZIKA VIRUS, EMPHYSEMA, LOW SPERM COUNT, ENZYMA, COUGH, ULCER, ARTHRITIS, LEUKAEMIA, LYME DISEASE, ASTHMA, IMPOTENCE, BARENESS/INFERTILITY, WEAK ERECTION, PENIS ENLARGEMENT. AND SO ON.  

    ReplyDelete
  6. I'm so excited to share my testimonies about the Good Work of  Dr. Anuge who got me cured from herpes simplex virus (HSV1&2) with his herbs, I never thought that I would live on earth before the year runs out. I have been suffering from herpes, I had spent a lot of money going from one Hospital to another looking for a way to get rid of this disease, the hospital has been my home everyday residence. Constant checks up have been my hobby not until this fateful day, I was searching through the internet, I saw a testimony on how @Dr_anuge7 helped someone in curing his herpes disease using his healing Herbs, quickly I copied his email just to give him a test I spoke to him, he told me that he is going to provide the herbal cure to me, which he did, after I received his herbs and I took it as instructed. I was cured permanently from herpes. My herpes disease was gone. so I decided to share my testimony, that nothing is impossible with God, God used a man to heal me. No matter what you are passing through, no matter how deadly the sickness is and no matter what the situation is God that did mine is still going to do yours, people suffering from herpes, brain tumor, kidney disease, pcos, AIDS, ALS, copd, asthma, arthritis, herpes, Cancer, Hpv, any kind of disease, you can reach him now via ? Gmail address: dranuge@gmail.com or whatsapp  +2348164866838

    ReplyDelete
  7. I'm so excited to share my testimonies about the Good Work of  Dr. Anuge who got me cured from herpes simplex virus (HSV1&2) with his herbs, I never thought that I would live on earth before the year runs out. I have been suffering from herpes, I had spent a lot of money going from one Hospital to another looking for a way to get rid of this disease, the hospital has been my home everyday residence. Constant checks up have been my hobby not until this fateful day, I was searching through the internet, I saw a testimony on how @Dr_anuge7 helped someone in curing his herpes disease using his healing Herbs, quickly I copied his email just to give him a test I spoke to him, he told me that he is going to provide the herbal cure to me, which he did, after I received his herbs and I took it as instructed. I was cured permanently from herpes. My herpes disease was gone. so I decided to share my testimony, that nothing is impossible with God, God used a man to heal me. No matter what you are passing through, no matter how deadly the sickness is and no matter what the situation is God that did mine is still going to do yours, people suffering from herpes, brain tumor, kidney disease, pcos, AIDS, ALS, copd, asthma, arthritis, herpes, Cancer, Hpv, any kind of disease, you can reach him now via ? Gmail address: dranuge@gmail.com or whatsapp  +2348164866838

    ReplyDelete
  8. Good day. I'm Mrs Lynn and I want to quickly share something with you. As of January 2019, I was having issues with my health and I was forced to go to the hospital. Only to be diagnosed of cancer. And as if that wasn't bad enough, it was prostate cancer. I felt like my greatest fear has finally caught up with me. I lived life like I wasn't existing no more. I was depressed, frustrated and most a times I contemplated suicide. Then one day while going through the internet checking for ways to get cured because I was getting sick of the too many drugs I was taking I stumbled across a post about a doctor called DR.Ehiaguna. I quickly messaged him because I was really desperate. I got a reply from DR.Ehiaguna and he told me what to do. And I did exactly as I was told and under two week I noticed changes. I could do things I normally found to be difficult and today I am cancer free and I will be forever grateful to doctor DR.Ehiaguna because he saved my life. Are you a cancer patient? Do you want to live a cancer free life? Then leave doctor Ehiaguna a message drehiaguna@gmail.com also WhatsApp him +2348073908953 and you'll be glad you did And He also have herbs medicine to cured the following diseases; eczema,urethra wart,chronic problems.Herpes, Cancer, Als,Hepatitis, Diabetes, HPV,Infections,ulcer  ETC

    ReplyDelete
  9. There are some natural remedies that can be used in the prevention and eliminate diabetes totally. However, the single most important aspect of a diabetes control plan is adopting a wholesome lifestyle Inner Peace, Nutritious and Healthy Diet, and Regular Physical Exercise. A state of inner peace and self-contentment is essential to enjoying a good physical health and overall well-being. The inner peace and self contentment is a just a state of mind.People with diabetes diseases often use complementary and alternative medicine. I diagnosed diabetes in 2010. Was at work feeling unusually tired and sleepy. I borrowed a cyclometer from a co-worker and tested at 760. Went immediately to my doctor and he gave me prescriptions like: Insulin ,Sulfonamides,Thiazolidinediones but Could not get the cure rather to reduce the pain but bring back the pain again. i found a woman testimony name Comfort online how Dr Akhigbe cure her HIV  and I also contacted the doctor and after I took his medication as instructed, I am now completely free from diabetes by doctor Akhigbe herbal medicine.So diabetes patients reading this testimony to contact his email     drrealakhigbe@gmail.com   or his Number   +2348142454860   He also use his herbal herbs to diseases like:SPIDER BITE, SCHIZOPHRENIA, LUPUS,EXTERNAL INFECTION, COMMON COLD, JOINT PAIN, EPILEPSY,STROKE,TUBERCULOSIS ,STOMACH DISEASE. ECZEMA, GOUT, PROGENITOR, EATING DISORDER, LOWER RESPIRATORY INFECTION,  DIABETICS,HERPES,HIV/AIDS, ;ALS,  CANCER , TUMOR MENINGITIS, WAIST PAIN ,HEPATITIS A AND B,ASTHMA, HEART DISEASE, CHRONIC DISEASE. NAUSEA VOMITING OR DIARRHEA,KIDNEY DISEASE. HEARING  LOSSDr Akhigbe is a good man and he heal anybody that comes to him. here is email    drrealakhigbe@gmail.com    and his Number +2349010754824

    ReplyDelete
  10. Great questionnaire on these vital digestive tract parts!

    ReplyDelete
  11. My husband was diagnosed with early onset Parkinson's disease at 57.his symptoms were shuffling of feet,slurred speech, low volume speech, degradation of hand writing, horrible driving skills, right arm held at 45 degree angle, things were tough for me, but now he finally free from the disease with the help of total cure ultimate health home, he now walks properly and all symptoms has reversed, he had trouble with balance especially at night, getting into the shower and exiting it is difficult,getting into bed is also another thing he finds impossible.we had to find a better solution for his condition which has really helped him a lot,the biggest helped we had was ultimatehealthhome they walked us through the proper steps,am highly recommended this ultimatehealthhome@gmail.com to anyone who needs help.

    ReplyDelete

  12. i was diagnosed of parkinson disease 5 years ago,i started azilect then mirapex as the disease progressed in frebuary last year,and i started on parkinson disease herbal treatment from Ultimate Life Clinic,few months into the treatment i made a significant recovery,almost all my symptoms are gone,great improvement with my movement and belance,it been a year and life has been so good for me,contact them through there website www.ultimatelifeclinic.com

    ReplyDelete

Related Posts Plugin for WordPress, Blogger...
Related Posts Plugin for WordPress, Blogger...